SlideShare a Scribd company logo
1 of 51
Download to read offline
TEST BANK FOR Advanced Assessment Interpreting
Findings and Formulating Differential Diagnoses
4th Edition Goolsby Chapters 1 - 22 | Complete
TABLE OF CONTENTS
 Chapter 1. Assessment and Clinical Decision Making: An Overview
 Chapter 2. Genomic Assessment: Interpreting Findings and Formulating Differential Diagnoses
 Chapter 3. Skin
 Chapter 4. Head, Face, and Neck
 Chapter 5. The Eye
 Chapter 6. Ear, Nose, Mouth, and Throat
 Chapter 7. Cardiac and Peripheral Vascular Systems
 Chapter 8. Respiratory System
 Chapter 9. Breasts
 Chapter 10. Abdomen
 Chapter 11. Genitourinary System
 Chapter 12. Male Reproductive System
 Chapter 13. Female Reproductive System
 Chapter 14. Musculoskeletal System
 Chapter 15. Neurological System
 Chapter 16. Nonspecific Complaints
 Chapter 17. Psychiatric Mental Health
 Chapter 18. Pediatric Patients
 Chapter 19. Pregnant Patients
 Chapter 20. Assessment of the Transgender or Gender Diverse Adult
 Chapter 21. Older Patients
 Chapter 22. Persons With Disabilities
 Chapter 1. Assessment and Clinical Decision Making: An Overview
Multiple Choice
Identify the choice that best completesthe statement or answers the question.
1. Which type of clinical decision-making is most reliable?
A. Intuitive
B. Analytical
C. Experiential
D. Augenblick
2. Which of the following is false? To obtain adequate history, health-care providers must be:
A. Methodical and systematic
B. Attentive to the patient’s verbal and nonverbal language
C. Able to accurately interpret the patient’s responses
D. Adept at reading into the patient’s statements
3. Essential parts of a health historyinclude all of the following except:
A. Chief complaint
B. Historyof the present illness
C. Current vital signs
D. All of the above are essential history components
4. Which of the following is false? While performing the physical examination, the examiner must be able to:
A. Differentiate between normal and abnormal findings
B. Recall knowledge of a range of conditions and their associated signs and symptoms
C. Recognize how certain conditions affect the response to other conditions
D. Foresee unpredictable findings
5. The following is the least reliable source of information for diagnostic statistics:
A. Evidence-based investigations
B. Primaryreports of research
C. Estimation based on a provider’s experience
D. Published meta-analyses
6. The following can be used to assist in sound clinical decision-making:
A. Algorithm published in a peer-reviewed journal article
B. Clinical practice guidelines
C. Evidence-based research
D. All of the above
7. If a diagnostic study has high sensitivity, this indicates a:
A. High percentage of persons with the given condition will have an abnormal result
B. Low percentage of persons with the given condition will have an abnormal result
C. Low likelihood of normal result in persons without a given condition
D. None of the above
8. If a diagnostic study has high specificity, this indicates a:
A. Low percentage of healthy individuals will show a normal result
B. High percentage of healthyindividuals will show a normal result
C. High percentage of individuals with a disorder will show a normal result
D. Low percentage of individuals with a disorder will show an abnormal result
9. Alikelihood ratio above 1 indicates that a diagnostic test showing a:
A. Positive result is strongly associated with the disease
B. Negative result is strongly associated with absence of the disease
C. Positive result is weakly associated with the disease
D. Negative result is weakly associated with absence of the disease
10. Which of the following clinical reasoning tools is defined as evidence-based resource based on mathematical modeling
to express the likelihood of a condition in select situations, settings, and/or patients?
A. Clinical practice guideline
B. Clinical decision rule
C. Clinical algorithm
Chapter 1: Clinical reasoning, differential diagnosis, evidence-based practice, and symptom ana
Answer Section
MULTIPLE CHOICE
1. ANS: B
Croskerry (2009) describes two major types of clinical diagnostic decision-making: intuitive and analytical. Intuitive decision-
making (similar to Augenblink decision-making) is based on the experience and intuition of the clinician and is less reliable and
paired with fairly common errors. In contrast, analytical decision-making is based on careful consideration and has greater reliability
with rare errors.
PTS: 1
2. ANS: D
To obtain adequate history, providers must be well organized, attentive to the patient’s verbal and nonverbal language, and able to
accurately interpret the patient’s responses to questions. Rather than reading into the patient’s statements, they clarify any areas of
uncertainty.
PTS: 1
3. ANS: C
Vital signs are part of the physical examination portion of patient assessment, not part of the health history.
PTS: 1
4. ANS: D
While performing the physical examination, the examiner must be able to differentiate between normal and abnormal findings, recall
knowledge of a range of conditions, including their associated signs and symptoms, recognize how certain conditions affect the
response to other conditions, and distinguish the relevance of varied abnormal findings.
PTS: 1
5. ANS: C
Sources for diagnostic statistics include textbooks, primary reports of research, and published meta-analyses. Another source of
statistics, the one that has been most widelyused and available for application to the reasoning process, is the estimation based on a
provider’s experience, although these are rarely accurate. Over the past decade, the availability of evidence on which to base clinical
reasoning is improving, and there is an increasing expectation that clinical reasoning be based on scientific evidence.
Evidence-based statistics are also increasingly being used to develop resources to facilitate clinical decision-making.
PTS: 1
6. ANS: D
To assist in clinical decision-making, a number of evidence-based resources have been developed to assist the clinician. Resources,
such as algorithms and clinical practice guidelines, assist in clinical reasoning when properly applied.
PTS: 1
7. ANS: A
The sensitivity of a diagnostic study is the percentage of individuals with the target condition who show an abnormal, or positive, result.
A high sensitivity indicates that a greater percentage of persons with the given condition will have an abnormal result.
PTS: 1
8. ANS: B
The specificity of a diagnostic study is the percentage of normal, healthy individuals who have a normal result. The greater the
specificity, the greater the percentage of individuals who will have negative, or normal, results if they do not have the target
condition.
PTS: 1
9. ANS: A
The likelihood ratio is the probability that a positive test result will be associated with a person who has the target condition and a
negative result will be associated with a healthy person. A likelihood ratio above 1 indicates that a positive result is associated with the
disease; a likelihood ratio less than 1 indicates that a negative result is associated with an absence of the disease.
PTS: 1
10. ANS: B
Clinical decision (or prediction) rules provide another support for clinical reasoning. Clinical decision rules are evidence-based
resources that provide probabilistic statements regarding the likelihood that a condition exists if certain variables are met with
regard to the prognosis of patients with specific findings. Decision rules use mathematical models and are specific to certain
situations, settings, and/or patient characteristics.
PTS: 1
Chapter 2. Evidence-based health screening
Multiple Choice
Identify the choice that best completes the statement or answers the question.
1. The first step in the genomic assessment of a patient is obtaining information regarding:
A. Family history
B. Environmental exposures
C. Lifestyle and behaviors
D. Current medications
2. An affected individual who manifests symptoms of a particular condition through whom a family with a genetic
disorder is ascertained is called a(n):
A. Consultand
B. Consulband
C. Index patient
D. Proband
3. An autosomal dominant disorder involves the:
A. X chromosome
B. Y chromosome
C. Mitochondrial DNA
D. Non-sex chromosomes
4. To illustrate a union between two second cousin family members in a pedigree, draw:
A. Arrows pointing tothe male and female
B. Brackets around the male and female
C. Double horizontal lines between the male and female
D. Circles around the male and female
5. To illustrate two family members in an adoptive relationship in a pedigree:
A. Arrows are drawn pointing to the male and female
B. Brackets are drawn around the male and female
C. Double horizontal lines are drawn between the male and female
D. Circles are drawn around the male and female
6. When analyzing the pedigree for autosomal dominant disorders, it is common to see:
A. Several generations of affected members
B. Many consanguineous relationships
C. More members of the maternal lineage affected than paternal
D. More members of the paternal lineage affected than maternal
7. In autosomal recessive (AR) disorders, individuals need:
A. Onlyone mutated gene on the sex chromosomes to acquire the disease
B. Onlyone mutated gene to acquire the disease
C. Two mutated genes to acquire the disease
D. Two mutated genes to become carriers
8. In autosomal recessive disorders, carriers have:
A. Two mutated genes; one from each parent that cause disease
B. A mutation on a sex chromosome that causes a disease
C. Asingle gene mutation that causes the disease
D. One copyof a gene mutation but not the disease
9. With an autosomal recessive disorder, it is important that parents understand that if theyboth carry a mutation, the
following are the risks to each of their offspring (each pregnancy):
A. 50% chance that offspring will carry the disease
B. 10% chance of offspring affected bydisease
10. A woman with an X-linked dominant disorder will:
C. 25% chance children will carrythe disease
D. 10% chance children will be disease free
A. Not be affected bythe disorder herself
B. Transmit the disorder to 50 % of her offspring (male or female)
C. Not transmit the disorder to her daughters
D. Transmit the disorder to onlyher daughters
11. In creating your female patient’s pedigree, you note that she and both of her sisters were affected by the same genetic
disorder. Although neither of her parents had indications of the disorder, her paternal grandmother and her paternal
grandmother’s two sisters were affected by the same condition. This pattern suggests:
A. Autosomal dominant disorder
B. Chromosomal disorder
C. Mitochondrial DNA disorder
D. X-linked dominant disorder
12. A woman affected with an X-linked recessive disorder:
A. Has one X chromosome affected bythe mutation
B. Will transmit the disorder to all of her children
C. Will transmit the disorder to all of her sons
D. Will not transmit the mutation to anyof her daughters
13. Which of the following are found in an individual with aneuploidy?
A. An abnormal number of chromosomes
B. An X-linked disorder
C. Select cells containing abnormal-appearing chromosomes
D. An autosomal recessive disorder
14. The pedigree of a family with a mitochondrial DNA disorder is unique in that:
A. None of the female offspring will have the disease
B. All offspring from an affected female will have disease
C. None of the offspring of an affected female will have the disease
D. All the offspring from an affected male will have disease
15. Which population is at highest risk for the occurrence of aneuploidy in offspring?
A. Mothers younger than 18
B. Fathers younger than 18
C. Mothers over age 35
D. Fathers over age 35
16. Approximately what percentage of cancers is due to a single-gene mutation?
A. 50% to 70%
B. 30% to 40%
C. 20% to 25%
D. 5% to 10%
17. According to the Genetic Information Nondiscrimination Act (GINA):
A. NPs should keep all genetic information of patients confidential
B. NPs must obtain informed consent prior to genetic testing of all patients
C. Employers cannot inquire about an employee’s genetic information
D. All of the above
18. The leading causes of death in the United States are due to:
A. Multifactorial inheritance
B. Single gene mutations
C. X-linked disorders
D. Aneuploidy
19. Which of the following would be considered a “red flag” that requires more investigation in a patient assessment?
A. Colon cancer in family member at age 70
20. When patients express variable forms of the same hereditarydisorder, this is due to:
B. Breast cancer in family member at age 75
C. Myocardial infarction in familymember at age 35
D. All of the above
A. Penetrance
B. Aneuploidy
C. De novo mutation
D. Sporadic inheritance
21. Your 2-year-old patient shows facial features, such as epicanthal folds, up-slanted palpebral fissures, single transverse
palmar crease, and a low nasal bridge. These arereferred to as:
A. Variable expressivity related to inherited disease
B. Dysmorphic features related to genetic disease
C. De novo mutations of genetic disease
D. Different penetrant signs of genetic disease
22. In order to provide a comprehensive genetic history of a patient, the NP should:
A. Ask patients to complete a family history worksheet
B. Seek out pathology reports related to the patient’s disorder
C. Interview family members regarding genetic disorders
D. All of the above
1. 2. Evidence-based health screening
Answer Section
MULTIPLE CHOICE
1. ANS: A
A critical first step in genomic assessment, including assessment of risk, is the use of family history. Family history is considered the
first genetic screen (Berry & Shooner 2004) and is a critical component of care because it reflects shared genetic susceptibilities,
shared environment, and common behaviors (Yoon, Scheuner, & Khoury2003).
PTS: 1
2. ANS: D
A proband is defined as the affected individual who manifests symptoms of a particular condition through whom a family with a
genetic disorder is ascertained (Pagon et al. 1993–2013). The proband is the affected individual that brings the family to medical
attention.
PTS: 1
3. ANS: D
Autosomal dominant (AD) inheritance is a result of a gene mutation in one of the 22 autosomes.
PTS: 1
4. ANS: C
A consanguineous family is related by descent from a common ancestry and is defined as a “union between two individuals who are
related as second cousins or closer” (Hamamy 2012). Consanguinity, if present in the family history, is portrayed using two horizontal
lines to establish the relationship between the male and female partners.
PTS: 1
5. ANS: B
For adopted members of the family, use brackets as the appropriate standardized pedigree symbol ([e.g., brackets]).
PTS: 1
6. ANS: A
Pedigrees associated with autosomal dominant (AD) disorders typically reveal multiple affected family members with the disease or
syndrome. When analyzing the pedigree for AD disorders or syndromes, it is common to see a “vertical” pattern denoting several
generations of affected members.
PTS: 1
7. ANS: C
In autosomal recessive (AR) disorders, the offspring inherits the condition by receiving one copy of the gene mutation from each of
the parents. Autosomal recessive disorders must be inherited through both parents (Nussbaum et al. 2007). Individuals who have an
AR disorder have two mutated genes, one on each locus of the chromosome. Parents of an affected person are called carriers because
each carries one copy of the mutation on one chromosome and a normal gene on the other chromosome. Carriers typically are not
affected bythe disease.
PTS: 1
8. ANS: D
Individuals who have an AR disorder have two mutated genes, one on each allele of the chromosome. Parents of an affected person
are called carriers because each parent carries one copy of the mutation on one chromosome and a normal gene on the other
chromosome. Carriers typically are not affected by the disease. In pedigrees with an AR inheritance patterns, males and females will
be equally affected because the gene mutation is on an autosome.
PTS: 1
9. ANS: A
It is important that parents understand that if they both carry a mutation, the risk to each of their offspring (each pregnancy) is an
independent event: 25% disease free, 25% affected, and 50% carrier.
PTS: 1
10. ANS: B
Everyone born with an X-linked dominant disorder will be affected with the disease. Transmission of the disorder to the next
generation varies bygender, however. A woman will transmit the mutation to 50% of all her offspring (male or female).
PTS: 1
11. ANS: D
A man with an X-linked dominant disorder will transmit the mutation to 100% of his daughters (they receive his X chromosome)
and none of his sons (they receive his Y chromosome). The pedigree of a family with an X-linked dominant disorder would reveal
all the daughters and none of the sons affected with the disorder if the father has an X-linked disorder.
PTS: 1
12. ANS: C
An X-linked recessive disorder means that in a woman, both X chromosomes must have the mutation if she is to be affected. Because
males have onlyone copy of the X chromosome, they will be affected if their X chromosome carries the mutation.
PTS: 1
13. ANS: A
An individual with an abnormal number of chromosomes has a condition called aneuploidy, which is frequently associated with mental
problems or physical problems or both (Jorde, Carey, & Bamshad 2010; Nussbaum et al. 2007).
PTS: 1
14. ANS: B
Mitochondrial DNA is inherited from the ovum and, therefore, from the mother. The pedigree of a family with a mitochondrial
DNA disorder is unique in that all offspring (regardless of gender) of an affected female will have the disease, and none of the
offspring from an affected male will have the disease.
PTS: 1
15. ANS: C
Some individuals or couples have unique identifiable risks that should be discussed prior to conception whenever possible. For
example, women who will be 35 years of age or older at delivery (advanced maternal age) are at increased risk for aneuploidy.
PTS: 1
16. ANS: D
The majority of cancers are sporadic or multifactorial due to a combination of genetic and environmental factors; however,
approximately5% to 10% of all cancers are due to a single-gene mutation (Garber & Offit 2005).
PTS: 1
17. ANS: D
On May 21, 2008, President George W. Bush signed the Genetic Information Nondiscrimination Act (GINA) to protect Americans
against discrimination based upon their genetic information when it comes to health insurance and employment, paving the way for
patient personalized genetic medicine without fear of discrimination (National Human Genome Research Institute 2012).
PTS: 1
18. ANS: A
Most disease-causing conditions are not due to a single-gene disorder but are due to multifactorial inheritance, a result of genomics
and environmental or behavioral influences. In fact, the leading causes of mortality in the United States—heart disease,
cerebrovascular disease, diabetes, and cancer—are all multifactorial. Most congenital malformation, hypertension, arthritis, asthma,
obesity, epilepsy, Alzheimer’s, and mental health disorders are also multifactorial.
PTS: 1
19. ANS: C
Early onset cancer syndromes, heart disease, or dementia are red flags that warrant further investigation regarding hereditary
disorders.
PTS: 1
20. ANS: A
Some disorders have a range of expression from mild to severe. This variability is referred to as the penetrance of genetic disease.
For example, patients with neurofibromatosis (NF1), an AD disorder of the nervous system, may manifest with many forms of the
disease. For instance, some patients with NF1 may have mild symptoms, like café-au-lait spots or freckling on the axillary or skin,
while others may have life-threatening spinal cord tumors or malignancy (Jorde, Carey, & Bamshad 2010; Nussbaum et al. 2007).
PTS: 1
21. ANS: B
Assessing for dysmorphic features may enable identification of certain syndromes or genetic or chromosomal disorders (Jorde,
Carey, & Bamshad 2010; Prichard & Korf 2008). Dysmorphology is defined as “the study of abnormal physical development”
(Jorde, Carey, & Bamshad 2010, 302).
PTS: 1
22. ANS: D
Asking the patient to complete a family history worksheet prior to the appointment saves time in the visit while offering the patient
an opportunity to contribute to the collection of an accurate family history. Reviewing the family information can also help establish
family rapport while verifying medical conditions in individual family members. If a hereditary condition is being considered but
family medical information is unclear or unknown, requesting medical records and pathology or autopsy reports may be warranted.
PTS: 1
Chapter 3. Abdomen
Multiple Choice
Identify the choice that best completes the statement or answers the question.
1. When performing abdominal assessment, the clinician should perform examination techniques in the following order:
A. Inspection, palpation, percussion, and auscultation
B. Inspection, percussion, palpation, and auscultation
C. Inspection, auscultation, percussion, and palpation
D. Auscultation, palpation, percussion, and inspection
2. The clinician should auscultate the abdomen to listen for possible bruits of the:
A. Aorta
B. Renal artery
C. Iliac artery
D. All of the above
3. On abdominal examination, which of the following is assessed using percussion?
A. Liver
B. Kidneys
C. Pancreas
D. Esophagus
4. In abdominal assessment, a digital rectal examination is performed to assess for:
A. Hemorrhoids
B. Prostate size
C. Blood in stool
D. Ureteral stenosis
5. Rebound tenderness of the abdomen is a sign of:
A. Constipation
B. Peritoneal inflammation
C. Elevated venous pressure
D. Peritoneal edema
6. While assessingthe abdomen, the clinician deeplypalpates the left lower quadrant of the abdomen, and this causes pain
in the patient’s right lower abdomen. This is most commonlyindicative of:
A. Constipation
B. Diverticulitis
C. Appendicitis
D. Hepatitis
7. Your patient complains of severe right lower quadrant abdominal pain. To assess the patient for peritoneal
inflammation, the examiner should:
A. Percuss the right lower quadrant of the abdomen
B. Deeplypalpate the right lower quadrant of the abdomen
C. Auscultate the right lower quadrant for hyperactive bowel sounds
D. Strike the plantar surface of the patient’s heel while the patient is supine
8. Your patient is lying supine and you ask him to raise his leg while you place resistance against the thigh. The examiner
is testing the patient for:
A. Psoas sign
B. Obturator sign
C. Rovsing’s sign
D. Murphys’ sign
9. A patient is lying supine and the clinician deeplypalpates the right upper quadrant of the abdomen while the patient
inhales. The examiner is testing the patient for:
A. Psoas sign
B. Obturator sign
C. Rovsing’s sign
10. Your patient has abdominal pain, and it is worsened when the examiner rotates the patient’s right hip inward with the
knee bent and the obturator internus muscle is stretched. This is a sign of:
A. Diverticulitis
B. Cholecystitis
C. Appendicitis
D. Mesenteric adenitis
11. On abdominal examination as the clinician presses on the right upper quadrant to assess liver size, jugular vein
distension becomes obvious. Hepatojugular reflux is indicative of:
A. Acute hepatitis
B. Right ventricular failure
C. Cholecystitis
D. Left ventricular failure
12. Your patient demonstrates positive shifting dullness on percussion of the abdomen. This is indicative of:
A. Cholecystitis
B. Appendicitis
C. Ascites
D. Hepatitis
13. Your 44-year-old female patient complains of right upper quadrant pain. Her skin and sclera are yellow, and she has
hyperbilirubinemia and elevated liver enzymes. The clinician should suspect:
A. Acute pancreatitis
B. Biliary duct obstruction
C. Acute hepatitis
D. Atypical appendicitis
14. The most common cause of acute pancreatitis is:
A. Trauma
B. Hepatitis virus A
C. Hyperlipidemia
D. Alcohol abuse
15. Your patient with pancreatitis has a Ranson rule score of 8. The clinician should recognize that this is arisk of:
A. Pleural involvement
B. Alcoholism
C. High mortality
D. Bile duct obstruction
16. Your patient complains of left upper quadrant pain, fever, extreme fatigue, and spontaneous bruising. The clinician
should recognize that these symptoms are often related to:
A. Hematopoetic disorders
B. Hepatomegaly
C. Esophageal varices
D. Pleural effusion
17. A 16-year-old patient presents with sore throat, cervical lymphadenopathy, fever, extreme fatigue, and left upper
quadrant pain. The physical examination reveals splenomegaly. The clinician should recognize the probability of:
A. Bacterial endocarditis
B. Infectious mononucleosis
C. Pneumonia with pleural effusion
D. Pancreatic cancer
18. Your patient complains of lower abdominal pain, anorexia, extreme fatigue, unintentional weight loss of 10 pounds in
last 3 weeks, and you find a positive hemoccult on digital rectal examination. Laboratory tests show iron deficiency anemia. The
clinician needs to consider:
A. Diverticulitis
B. Appendicitis
C. Colon cancer
D. Murphys’ sign
19. Which of the following is the most common cause of heartburn-type epigastric pain?
A. Decreased lower esophageal sphincter tone
B. Helicobacteria pylori infection of stomach
C. Esophageal spasm
D. Excess use of NSAIDs
20. A 22-year-old female enters the emergency room with complaints of right lower quadrant abdominal pain, which has
been worsening over the last 24 hours. On examination of the abdomen, there is a palpable mass and rebound tenderness over the
right lower quadrant. The clinician should recognize the importance of:
A. Digital rectal examination
B. Endoscopy
C. Ultrasound
D. Pelvic examination
21. The major sign of ectopic pregnancy is:
A. Sudden onset of severe epigastric pain
B. Amenorrhea with unilateral lower quadrant pain
C. Lower back and rectal pain
D. Palpable abdominal mass
22. When ruptured ectopic pregnancyis suspected, the following procedure is most important:
A. Culdocentesis
B. CT scan
C. Abdominal x-ray
D. Digital rectal examination
23. The majorityof colon cancers are located in the:
A. Transverse colon
B. Cecum
C. Rectosigmoid region
D. Ascending colon
24. The following symptom(s) in the patient’s history should raise the clinician’s suspicion of colon cancer:
A. Alternating constipation and diarrhea
B. Narrowed caliber of stool
C. Hematochezia
D. All of the above
25. A patient presents tothe emergencydepartment with nausea and severe, colickyback pain that radiates into the groin.
When asked to locate the pain, he points to the right costovertebral angle region. His physical examination is unremarkable. Which of
the following lab tests is most important for the diagnosis?
A. Urinalysis
B. Serum electrolyte levels
C. Digital rectal exam
D. Lumbar x-ray
26. Your 34-year-old female patient complains of a feeling of “heaviness” in the right lower quadrant, achiness, and
bloating. On pelvic examination, there is a palpable mass in the right lower quadrant. Urine and serum pregnancy tests are negative.
The diagnostic tool that would be most helpful is:
A. Digital rectal exam
B. Transvaginal ultrasound
C. Pap smear
D. Urinalysis
27. Your 54-year-old male patient complains of a painless “lump” in his lower left abdomen that comes and goes for the
past couple of weeks. When examining the abdomen, you should have the patient:
A. Lie flat and take a deep breath
B. Stand and bear down against your hand
C. Prepare for a digital rectal examination
D. Peptic ulcer disease
28. A nurse practitioner reports that your patient’s abdominal x-raydemonstrates multiple air-fluid levels in the bowel.
This is a diagnostic finding found in:
A. Appendicitis
B. Cholecystitis
C. Bowel obstruction
D. Diverticulitis
29. A 76-year-old patient presents to the emergency department with severe left lower quadrant abdominal pain, diarrhea,
and fever. On physical examination, you note the patient has a positive heel strike, and left lower abdominal rebound tenderness.
These are typical signs and symptoms of which of the following conditions?
A. Diverticulitis
B. Salpingitis
C. Inflammatory bowel disease
D. Irritable bowel syndrome
30. Which of the following conditions is the most common cause of nausea, vomiting, and diarrhea?
A. Viral gastroenteritis
B. Staphylococcal food poisoning
C. Acute hepatitis A
D. E.coli gastroenteritis
31. A patient presents tothe emergencydepartment with complaints of vomiting and abdominal pain. You note that the
emesis contains bile. On physical examination, there is diffuse tenderness, abdominal distension, and rushing, high-pitched bowel
sounds. Which of the following diagnoses would be most likely?
A. Gastric outlet obstruction
B. Small bowel obstruction
C. Distal intestinal blockage
D. Colonic obstruction
32. Your 5-year-old female patient presents to the emergency department with sore throat, vomiting, ear ache, 103 degree
fever, photophobia, and nuchal rigidity. She has an episode of projectile vomiting while you are examining her. The clinician should
recognize that the following should be done:
A. Abdominal x-ray
B. Fundoscopic examination
C. Lumbar puncture
D. Analysis of vomitus
33. A9-year-old boyaccompanied byhis mother reports that since he came home from summer camp, he has had fever,
nausea, vomiting, severe abdominal cramps and watery stools that contain blood and mucus. The clinician should recognize the
importance of:
A. Stool for ova and parasites
B. Abdominal x-ray
C. Stool for clostridium
D. Fecal occult blood test
34. A 56-year-old male complains of anorexia, changes in bowel habits, extreme fatigue, and unintentional weight loss. At
times he is constipated and other times he has episodes of diarrhea. His physical examination is unremarkable. It is important for the
clinician to recognize the importance of:
A. CBC with differential
B. Stool culture and sensitivity
C. Abdominal x-ray
D. Colonoscopy
35. A 20-year-old engineering student complains of episodes of abdominal discomfort, bloating, and episodes of diarrhea.
The symptoms usually occur after eating, and pain is frequently relieved with bowel movement. She is on a “celiac diet” and the
episodic symptoms persist. Physical examination and diagnostic tests are negative. Colonoscopy is negative for any abnormalities.
This is a history and physical consistent with:
A. Inflammatory bowel disease
B. Irritable bowel syndrome
D. Lie in a left lateral recumbent position
C. Laxative abuse
D. Norovirus gastroenteritis
36. A 78-year-old female patient is suffering from heart failure, GERD, diabetes, and depression. She presents with
complaints of frequent episodes of constipation. Her last bowel movement was 1 week ago. Upon examination, you palpate a hard
mass is the left lower quadrant of the abdomen. You review her list of medications. Which of the following of her medications
cause constipation?
A. Digitalis (Lanoxin)
B. Amlodipine (Norvasc)
C. Sertraline (Zoloft)
D. Metformin (Glucophage)
37. You are examining a 55-year-old female patient with a historyof alcohol abuse. She complains of anorexia, nausea,
pruritus, and weight loss over the last month. On physical examination, you note yellow hue of the skin and sclera. Which of the
following physical examination techniques is most important?
A. Scratch test
B. Heel strike
C. Digital rectal examination
D. Pelvic examination
38. You observe Charcot’s triad of sign and symptoms in a patient under your care. This is commonlyseen in which of the
following disorders?
A. Cirrhosis
B. Pancreatitis
C. Cholangitis
D. Portal hypertension
39. A 59-year-old patient with history of alcohol abuse is admitted for hematemesis. On physical examination, you note
ascites and caput medusa. Alikelycause for the hematemesis is:
A. Peptic ulcer disease
B. Barrett’s esophagus
C. Pancreatitis
D. Esophageal varices
40. A 16-year-old female with anorexia and bulimia is admitted for hematemesis. She admits to inducing vomiting often.
On physical examination, you note pallor, BMI less than 15, and hypotension. Alikelyreason for hematemesis is:
A. Mallory-Weiss tear
B. Cirrhosis
C. Peptic ulcer disease
D. Esophageal varies
41. An 82-year-old female presents to the emergencydepartment with epigastric pain and weakness. She admits to having
dark, tarry stools for the last few days. She reports a long history of pain due to osteoarthritis. She self-medicates daily with
ibuprofen, naprosyn, and aspirin for joint pain. On physical examination, she has orthostatic hypotension and pallor. Fecal occult
blood test is positive. A likely etiology of the patient’s problem is:
A. Mallory-Weiss tear
B. Esophageal varices
C. Gastric ulcer
D. Colon cancer
42. A 48-year-old male presents to the clinic with complaints of anorexia, nausea, weakness, and unintentional weight loss
over the last few weeks. On physical examination, the patient has jaundice of the skin as well as sclera and a palpable mass in the
epigastric region. In addition to CBC and bilirubin levels, all of the following tests would be helpful except:
A. Liver enzymes
B. Amylase
C. Lipase
D. Uric acid
43. Your 66-year-old male patient complains of weakness, fatigue, chronic constipation for the last month, and dark stools.
On CBC, his results show iron deficiency anemia. Colon cancer is diagnosed. Which of the following laboratory tests is used to
follow progress of colon cancer?
A. Alpha fetoprotein (AFP)
B. Carcinogenic embryonic antigen (CEA)
C. Carcinoma antigen 125 (CA-125)
D. Beta-human chorionic gonadotropin (beta HCG)
44. Your patient is a 33-year-old female gave birth last week. She complains of constipation, rectal pain, and itching. She
reports bright red blood on the toilet tissue. The clinician should recognize the need for:
A. Digital rectal exam
B. CEA blood test
C. Colonoscopy
D. Fecal occult blood test
Chapter 3. Abdomen
Answer Section
MULTIPLE CHOICE
1. ANS: C
The abdominal examination begins with inspection, followed by auscultation, percussion, and palpation. Light palpation should
precede deep palpation. Auscultating before percussion or palpation allows the examiner to listen to the abdominal sounds
undisturbed. Moreover, if pain is present, it is best to leave palpation until last and to gather other data before possibly causing the
patient discomfort.
PTS: 1
2. ANS: D
Perform auscultation before palpation so as to hear unaltered bowel sounds. Listen for bruits over the aorta and the iliac, renal, and
femoral arteries.
PTS: 1
3. ANS: A
The purpose of liver percussion is to measure the liver size. The technique used to percuss theliver is as follows:
1. Starting in the midclavicular line at about the 3rd intercostal space, lightly percuss and move down.
2. Percuss inferiorlyuntil dullness denotes the liver's upper border (usuallyat fifth intercostal space inMCL).
3. Resume percussion from below the umbilicus on the midclavicular line in an area of tympany.
4. Percuss superiorlyuntil dullness indicates the liver's inferior border.
5. Measure span in centimeters. Normal liver span: clinically estimated at midclavicular line: 6-12 cm and midsternal line: 4-8 cm.
PTS: 1
4. ANS: D
A digital rectal examination is included in the abdominal examination. Note skin changes or lesions in the perianal region or the
presence of external hemorrhoids. Insert the gloved index finger into the anus with the patient either leaning over or side-lying on the
examination table, and note any internal hemorrhoids or fissures. Check the stool for occult blood. For males, the rectal
examination is necessary for direct examination of the prostate. Ureteral stenosis is detected by angiographt.
PTS: 1
5. ANS: B
Rebound tenderness is tested by slowly pressing over the abdomen with your fingertips, holding the position until pain subsides or
the patient adjusts to the discomfort, and then quickly removing the pressure. Rebound pain, a sign of peritoneal inflammation, is
present if the patient experiences a sharp discomfort over the inflamed site when pressure is released.
PTS: 1
6. ANS: C
Appendicitis is suggested by a positive Rovsing’s sign. This sign is positive when there is referred rebound pain in the right lower
quadrant when the examiner presses deeplyin the left lower quadrant and then quickly releases the pressure.
PTS: 1
7. ANS: D
Ask the patient to stand with straight legs and to raise up on toes. Then ask the patient to relax, allowing the heel to strike the floor,
thus jarring the body. A positive heel strike is indicative of appendicitis and peritoneal irritation. Alternatively, strike the plantar
surface of the heel with your fist while the patient rests supine on the examination table.
PTS: 1
8. ANS: A
To examine the patient for appendicitis, the clinician can test the patient for psoas sign. This is done in the following manner: Place a
hand on the patient’s thigh just above the knee and ask the patient to raise the thigh against your hand. This contracts the psoas
muscle and produces pain in patients with an inflamed appendix.
PTS: 1
9. ANS: D
Murphy’s Sign is elicited by deeply palpating the right upper quadrant of the abdomen. Pain is present on deep inspiration when an
inflamed gallbladder is palpated by pressing the fingers under the rib cage. Murphy’s sign is positive in cholecystitis.
PTS: 1
10. ANS: C
A positive obturator sign indicates appendicitis. Pain is elicited by inward rotation of the right hip with the knee bent so that the
obturator internus muscle is stretched.
PTS: 1
11. ANS: B
Hepatojugular reflux is elicited by applying firm, sustained hand pressure to the abdomen in the midepigastric region while the
patient breathes regularly. Observe the neck for elevation of the jugular venous pressure (JVP) with pressure of the hand and a
sudden drop of the JVP when the hand pressure is released. Hepatojugular reflux is exaggerated in right heart failure.
PTS: 1
12. ANS: C
To assess the patient for ascites, test for shifting of the peritoneal fluid to the dependent side by rolling the patient side to side and
percussing for dullness on the dependent side of the abdomen.
PTS: 1
13. ANS: B
In cholecystitis, acute colicky pain is localized in the RUQ and is often accompanied by nausea and vomiting. Murphy’s sign is
frequently present. Fever is low grade, and the increase in neutrophilic leukocytes in the blood is slight. Acute cholecystitis improves
in 2 to 3 days and resolves within a week; however, recurrences are common. If acute cholecystitis is accompanied by jaundice and
cholestasis (arrest of bile excretion), suspect common duct obstruction.
PTS: 1
14. ANS: D
Biliary tract disease and alcoholism account for 80% or more of the pancreatitis admissions. Other causes include hyperlipidemia,
drugs, toxins, infection, structural abnormalities, surgery, vascular disease, trauma, hyperparathyroidism and hypercalcemia, renal
transplantation, and hereditary pancreatitis. The most common cause of pancreatitis is alcohol abuse.
PTS: 1
15. ANS: C
The Ranson rule uses a score determined by MRI results, with an index possible range of 0 to 10. A categorization of patients
indicates the risk of both mortality and complication from pancreatitis. Patients at the low end of the index (1–3) are predicted to
have a low risk of mortality (3%) and complications (8%), whereas patients scoring at the high end (7–10) of the index are predicted
to have a higher incidence of mortality(17%) and/or complications (92%).
PTS: 1
16. ANS: A
LUQ pain can be associated with stomach or spleen disorders; however, it is often associated with causes that are outside the
abdomen. Hematopoietic malignancies, such as lymphomas and leukemias, and other hematologic disorders, such as
thrombocytopenia, polycythemia, myelofibrosis, and hemolyticanemia, often cause enlargement of the spleen, leading to LUQ pain.
In addition to questions about the specific characteristics of the pain, it is important to ask the patient about fever, unusual bleeding
or bruising, recent diagnosis of mononucleosis, fatigue, malaise, lymphadenopathy, cough, arthralgias, anorexia, weight loss, jaundice,
high blood pressure, and headache.
PTS: 1
17. ANS: B
Hypersplenism is secondary to other primary disorders, most commonly cytopenic hematologic disorders, such as lymphoma,
leukemia, thrombocytopenia, polycythemia, myelofibrosis, and haemolytic anemias. With the sore throat and cervical
lymphadenopathy, infection due to Epstein-Barr virus is common in adolescents. Infectious mononucleosis is an important disorder
to consider. Splenomegaly often occurs in infectious mononucleosis.
PTS: 1
18. ANS: C
A positive hemoccult on rectal examination may indicate an upper GI bleed or malignancy. Malignancy should also be suspected if
there is weight loss and/or a palpable abdominal mass.
PTS: 1
19. ANS: A
GERD is the most common organic cause of heartburn. GERD is caused by decreased lower esophageal sphincter (LES) tone. LES
control can be decreased by several medications (e.g., theophylline, dopamine, diazepam, calcium-channel blockers), foods and/or
beverages (caffeine, alcohol, chocolate, fatty foods), and tobacco use. When LES tone is lower than normal, secretions are allowed to
reflux into the esophagus, causing discomfort.
PTS: 1
20. ANS: D
A female with abdominal pain can have a GI or GU disorder or gynecologic problem. It is imperative to ask about the last menstrual
period (LMP) and about birth control methods in order to rule out ectopic pregnancy. A history of miscarriages and/or sexually
transmitted diseases (STDs) can give more clues for the risk of ectopic pregnancy. Safe sex practices and the number of sexual
partners can alert the practitioner to the risk for pelvic inflammatory disease. No complaint of lower abdominal pain in a female
should be evaluated without performing a pelvic examination.
PTS: 1
21. ANS: B
The most obvious sign of ectopic pregnancy is amenorrhea followed by spotting and sudden onset of severe lower quadrant pain. A
stat pregnancy test should be performed. There is tenderness on pelvic examination, and a pelvic mass may be palpated. Blood is
present in the cul-de-sac. Shock and hemorrhage occur if the pregnancy ruptures. Abdominal distention with peritoneal signs will
ensue. Immediate laparoscopy or laparotomy is indicated because this condition is life threatening.
PTS: 1
22. ANS: A
The most obvious sign of ectopic pregnancy is amenorrhea followed by spotting and sudden onset of severe lower quadrant pain. A
stat pregnancy test should be performed. The diagnosis of ectopic pregnancy can be made with urine human chorionic gonadotropin
(hCG) or stat serum hCG, pelvic ultrasound, and, if necessary, culdocentesis to detect blood in the cul-de-sac.
There is tenderness on pelvic examination, and a pelvic mass may be palpated. Blood is present in the cul-de-sac. Shock and
hemorrhage occur if the pregnancy ruptures. Abdominal distension with peritoneal signs will ensue. Immediate laparoscopy or
laparotomy is indicated because this condition is life threatening.
PTS: 1
23. ANS: C
Colorectal cancer is the second leading cause of death from malignancies in the United States. Over half are located in the
rectosigmoid region and are typically adenocarcinomas. Risk factors include a history of polyps, positive family history of colon
cancer or familial polyposis, ulcerative colitis, granulomatous colitis, and a diet low in fiber and high in animal protein, fat, and
refined carbohydrates.
PTS: 1
24. ANS: D
Colon cancer may be present for several years before symptoms appear. Complaints include fatigue, weakness, weight loss,
alternating constipation and diarrhea, a change in the caliber of stool, tenesmus, urgency, and hematochezia. Physical examination is
usually normal except in advanced disease, when the tumor can be palpated or hepatomegaly is present, owing to metastatic disease.
PTS: 1
25. ANS: A
Urinary calculi can occur anywhere in the urinary tract; therefore, pain can originate in the flank or kidney area and radiate into the
RLQ or LLQ and then to the suprapubic area as the stone attempts to move down the tract. The pain is severe, acute, and colicky
and may be accompanied by nausea and vomiting. If the stone becomes lodged at the ureterovesical junction, the patient will
complain of urgency and frequency. Blood will be present in the urine.
PTS: 1
26. ANS: B
Ovarian masses are often asymptomatic, but symptoms may include pressure-type pain, heaviness, aching, and bloating. Masses are
typically detected on pelvic examination. In advanced malignancies, ascites is often present. An elevated cancer antigen 125 (CA-125)
result indicates the likelihood that the mass is malignant. A transvaginal pelvic ultrasound has a higher diagnostic sensitivity than
transabdominal ultrasound. If diagnosis is unclear, CT, MRI, or PET scan can be performed. A laparoscopy or exploratory
laparotomy is necessary for staging, tumor debulking, and resection.
PTS: 1
27. ANS: B
In the majority of hernia cases, a history of heavy physical labor or heavy lifting can be elicited. Right or left lower quadrant pain that
may radiate into the groin or testicle is typical. The pain is usually dull or aching unless strangulated, in which case the pain is more
severe. The pain increases with straining, lifting, or movement of the lower extremities. Physical examination includes palpating the
femoral area and inguinal ring for bulging or tenderness. Ask the patient to bear down against your hand.
PTS: 1
28. ANS: C
The most common causes of mechanical obstruction are adhesions, almost exclusively in patients with previous abdominal surgery,
hernias, tumors, volvulus, inflammatory bowel disease (Crohn’s disease, colitis), Hirschsprung’s disease, fecal impaction, and
radiation enteritis. Initially, the patient complains of a cramping periumbilical pain that eventually becomes constant. Physical
examination reveals mild, diffuse tenderness without peritoneal signs, and possibly visible peristaltic waves. In early obstruction,
tinkles, rushes, and borborygmi can be heard. In late obstruction, bowel sounds may be absent. The diagnosis can be made with flat
and upright abdominal films looking for bowel distension and the presence of multiple air-fluid levels. CT or MRI may be necessary
for confirmation.
PTS: 1
29. ANS: A
Diverticular disease is prevalent in patients over 60 years of age. Since the sigmoid colon has the smallest diameter of any portion of
the colon, it is the most common site for the development of diverticula. Although the pain can be generalized, it is typically localized
to the left lower abdomen and is accompanied by tenderness, fever, and leukocytosis. Other symptoms can include constipation or
loose stools, nausea, vomiting, and positive stool occult blood. With diverticulitis, there is an increased risk of perforation, which
presents with a more dramatic clinical picture as a result of peritonitis. Look for signs of peritonitis, such as a positive heel strike test
and/or rebound tenderness.
PTS: 1
30. ANS: A
Viral gastroenteritis is the most common cause of nausea, vomiting, and diarrhea. At least 50% of cases of gastroenteritis as
foodborne illness are due to norovirus. Another 20% of cases, and the majority of severe cases in children, are due to rotavirus.
Other significant viral agents include adenovirus and astrovirus.
PTS: 1
31. ANS: B
The contents of the vomitus commonly vary according to the level of obstruction. Gastric outlet obstruction is associated with
emesis containing undigested food. Proximal small intestinal blockage is likely to be bile-stained. Distal intestinal blockage is more
likely to contain fecal matter. The degree of cramping and pain is often related to the proximity of the obstruction, so that
obstructions of the lower intestines may have less severe cramping, vomiting, and/or pain. Bowel sounds often are high pitched and
metallic sounding but may later become absent. Tenderness may be localized or diffuse. Distention as well as a succussion splash
may be present.
PTS: 1
32. ANS: C
The range of neurologic disorders that result in nausea and/or vomiting is broad. Included are meningitis, increased intracranial
pressure (ICP), migraines, a space-occupying lesion, and Ménière’s disorder. Central nervous system-related vomiting is often
projectile and may not be preceded by nausea. Papilledema may accompany increased ICP. Neurological deficits may be evident with
increased ICP, space-occupying lesions, and meningitis. Nuchal rigidity is a classic finding for meningitis.
PTS: 1
33. ANS: A
Parasites causing diarrhea usually enter the body through the mouth. They are swallowed and can remain in the intestine or burrow
through the intestinal wall and invade other organs. Certain parasites, most commonly Giardia lamblia, transmitted by fecally
contaminated water or food, can cause diarrhea, bloating, flatulence, cramps, nausea, anorexia, weight loss, greasy stools because of
its interference with fat absorption, and occasionallyfever. Symptoms usually occur about 2 weeks after exposure and
can last 2 to 3 months. Often, the symptoms are vague and intermittent, which makes diagnosis more difficult. Serial stool
samples for O&P should be ordered because a single sample may not reveal the offending parasite.
PTS: 1
34. ANS: D
The symptoms and severity of the diarrhea vary according to the underlying cause. The symptoms of carcinomas are generally
insidious. The diarrhea is mild and intermittent. Often malignancies are found on routine hemoccults, sigmoidoscopy, or
colonoscopy. There should be a high index of suspicion with unexplained weight loss or new-onset iron-deficiency anemia in a
patient over 40 years old.
PTS: 1
35. ANS: B
Irritable bowel syndrome (IBS) is a functional bowel disorder characterized by mild to severe abdominal pain, discomfort, bloating,
and alteration of bowel habits. The exact cause is unknown. In some cases, the symptoms are relieved by bowel movements.
Diarrhea or constipation may predominate, or they may be mixed (classified as IBS-D, IBS-C, or IBS-M, respectively). IBS may
begin after an infection (postinfectious, IBS-PI) or a stressful life event. IBS is a motility disorder involving the upper and lower GI
tracts that causes intermittent nausea, abdominal pain and distention, flatulence, pain relieved by defecation, diarrhea, and/or
constipation. Symptoms usually occur in the waking hours and may be worsened or triggered by meals. It is three times more
prevalent in women, accounts for more than half of all GI referrals, and is highly correlated with emotional factors, particularly
anxiety and stress.
PTS: 1
36. ANS: B
Medications that frequently cause constipation include:
- Analgesics/narcotics
- Antacids containing aluminum
- Anticonvulsants
- Antidepressants
- Antihypertensives (calcium-channel blockers, beta blockers)
- Antiparkinsonism agents
- Antispasmodics
- Calcium supplements
- Diuretics
- Iron supplements
- Sedatives/tranquilizers
PTS: 1
37. ANS: A
Cirrhosis develops with the replacement of normal liver tissue by regenerative, fibrotic nodules and may occur in the late phase of a
variety of disorders that damage the liver, such as alcohol toxicity. A patient may present with jaundice and describe an associated,
progressive pattern of pruritus, weakness, anorexia, nausea, and weight loss. Determine the size and consistency of the liver as well
as any tenderness. The scratch test is a method used to ascertain the location and size of a patient's liver during a physical
assessment. The scratch test uses auscultation to detect the differences in sound transmission through the abdominal cavity over
solid and hollow organs and spaces. After placing a stethoscope over the approximate location of a patient's liver, the examiner will
then scratch the skin of the patient's abdomen lightly, moving laterally along the liver border. When the liver is encountered, the
scratching sound heard in the stethoscope will increase significantly. In this manner, the size and shape of a patient's liver can be
ascertained.
PTS: 1
38. ANS: C
Occlusion of the common bile duct may occur with disorders of the gallbladder and/or bile duct, such as cholecystitis, cholelithiasis,
and cholangitis. All three conditions are generally accompanied by RUQ discomfort, anorexia, and nausea. Charcot’s triad, which
includes jaundice, RUQ pain, and fever/chills, is common to problems resulting in obstructions of the bile duct.
PTS: 1
39. ANS: D
Patients with portal hypertension may develop GI bleeding from varices of the esophagus, stomach, intestines, or other sites. Portal
hypertension is most commonly associated with cirrhosis, usually caused by alcohol abuse or hepatitis. Check for signs of liver
disease, including jaundice, cirrhosis, telangiectasia, hepatomegaly, and RUQ tenderness. Ascites occurs due to venous congestion.
Caput medusa is the distension of paraumbilical veins due to portal hypertension.
PTS: 1
40. ANS: A
Upper GI hemorrhage may result from a tear at the gastroesophageal junction, known as a Mallory-Weiss tear. A patient may
develop more than one tear. These tears are most common in alcoholic or bulimic patients following repeated episodes of vomiting
or severe retching. If a laceration/tear of the mucosa causes GI bleeding, the patient may demonstrate alterations in hemodynamic
status.
PTS: 1
41. ANS: C
Bleeding occurs after an area of gastric mucosal injury has ulcerated. Explore symptoms of epigastric and/or periumbilical
discomfort. Identify potential causes of gastric mucosal injury—the most common being NSAID use and stress. Many elderly
individuals self-medicate with over-the-counter aspirin preparations and various NSAIDs. Commonly, they use too many medications
that have side effects of gastric irritation.
PTS: 1
42. ANS: D
Primary or metastatic cancers of the liver and/or pancreas can cause obstructive hyperbilirubinemia and jaundice. Jaundice may be
the initial sign of a malignancy or may follow the development of other symptoms. Ask about associated symptoms, such as RUQ
discomfort, nausea, fever, back pain, weight loss, fatigue/weakness, and pruritus. None of these symptoms are specific to
malignancy; however, other causes of jaundice are less likely to be associated with weight loss. During the abdominal examination,
carefully palpate the area of the liver and the remainder of the abdomen, checking for masses or unexpected findings. In addition to
a CBC, liver functions, amylase, lipase, and bilirubin levels, abdominal CT and/or ultrasound should be ordered promptly.
PTS: 1
43. ANS: B
AFP can help diagnose and guide the treatment of liver cancer (hepatocellular carcinoma). CA-125 is the standard tumor marker used
to follow women during or after treatment for epithelial ovarian cancer (the most common type of ovarian cancer) as well as fallopian
tube cancer and primary peritoneal cancer. Serum beta HCG is a pregnancy marker. CEA is not used to diagnose or screen for
colorectal cancer, but it’s the preferred tumor marker to help predict outlook in patients with colorectal cancer. The higher the CEA
level at the time colorectal cancer is detected, the more likelyit is that the cancer is advanced.
PTS: 1
44. ANS: A
The most common cause of lower GI bleeding is hemorrhoids. The bleeding associated with hemorrhoids is usually evident as red
blood on the formed stool, in the toilet bowl, or on the toilet tissue following a bowel movement. Patients with hemorrhoids often
complain of rectal discomfort as well as the contributing factors for hemorrhoid development, including constipation.
Inspect the perianal rectal tissue. Anoscopy may be indicated. Perform a digital rectal examination to assess internal haemorrhoids.
PTS: 1
Chapter 4: Affective Changes
MULTIPLE CHOICE
1. When performing a physical assessment, the first technique the nurse will always
use is:
a. Palpation.
b. Inspection.
c. Percussion.
d. Auscultation.
ANS: B
The skills requisite for the physical examination are inspection, palpation, percussion, and
auscultation. The skills are performed one at a time and in this order (with the exception
of the abdominal assessment, during which auscultation takes place before palpation and
percussion). The assessment of each body system begins with inspection. A focused
inspection takes time and yields a surprising amount of information.
2. The nurse is preparing to perform a physical assessment. Which statement
is true about the physical assessment? The inspection phase:
a. Usually yields little information.
b. Takes time and reveals a surprising amount of information.
c. Maybe somewhat uncomfortable for the expert practitioner.
d. Requires a quick glance at the patients body systems before proceeding with palpation.
ANS: B
Afocused inspection takes time and yields a surprising amount of information.
Initially, the examiner may feel uncomfortable, staring at the person without
also doing something. A focused assessment is significantly more than a quick glance.
3. The nurse is assessing a patients skin during an office visit. What part of the hand
and technique should be used to best assess the patients skin temperature?
a. Fingertips; they are more sensitive to small changes in temperature.
b. Dorsal surface of the hand; the skin is thinner on this surface than on the palms.
c. Ulnar portion of the hand; increased blood supply in this area enhances temperature sens
Palmar surface of the hand; this surface is the most sensitive to temperature variations b
d.
ANS:B
of its increased nerve supply in this area.
The dorsa (backs) of the hands and fingers are best for determining temperature
because the skin is thinner on the dorsal surfaces than on the palms. Fingertips are best
for fine, tactile discrimination. The other responses are not useful for palpation.
lO M oA R cP S D| 1 2 2 6 3 4 2 3
4. Which of these techniques uses the sense of touch to assess texture, temperature,
moisture, and swelling when the nurse is assessing a patient?
a. Palpation
b. Inspection
c. Percussion
d. Auscultation
ANS: A
Palpation uses the sense of touch to assess the patient for these factors. Inspection
involves vision; percussion assesses through the use of palpable vibrations and audible
sounds; and auscultation uses the sense of hearing.
5. The nurse is preparing to assess a patients abdomen by palpation. How should the
nurse proceed?
Palpation of reportedly tender areas are avoided because palpation in these areas may ca
a. pain.
Palpating a tender area is quickly performed to avoid any discomfort that the patient ma
b. experience.
The assessment begins with deep palpation, while encouraging the patient to relax and to
c. deep breaths.
The assessment begins with light palpation to detect surface characteristics and to accust
d.
ANS:D
patient to beingtouched.
Light palpation is initially performed to detect any surface characteristics and to
accustom the person to being touched. Tender areas should be palpated last, not first.
6.The nurse would use bimanual palpation technique in which situation?
a. Palpating the thorax of an infant
b. Palpating the kidneys and uterus
c. Assessingpulsations and vibrations
d. Assessing the presence of tenderness and pain
ANS: B
Bimanual palpation requires the use of both hands to envelop or capture certain body
parts or organs such as the kidneys, uterus, or adnexa. The other situations are not
appropriate for bimanual palpation.
7. The nurse is preparing to percuss the abdomen of a patient. The purpose of the
percussion is to assess the of the underlying tissue.
a. Turgor
b. Texture
Downloaded byAnna Maina(annamurugijoe@gmail.com)
lO M oA R cP S D| 1 2 2 6 3 4 2 3
Downloaded byAnna Maina(annamurugijoe@gmail.com)
c. Density
d. Consistency
ANS: C
Percussion yields a sound that depicts the location, size, and density of the underlying
organ. Turgor and texture are assessed with palpation.
8. The nurse is reviewing percussion techniques with a newly graduated nurse. Which
technique, if used by the new nurse, indicates that more review is needed?
a. Percussing once over each area
b. Quickly lifting the striking finger after each stroke
c. Striking with the fingertip, not the finger pad
d. Using the wrist to make the strikes, not the arm
ANS: A
For percussion, the nurse should percuss two times over each location. The striking
finger should be quickly lifted because a resting finger damps off vibrations. The tip of
the striking finger should make contact, not the pad of the finger. The wrist must be
relaxed and is used to make the strikes, not the arm.
9. When percussing over the liver of a patient, the nurse notices a dull sound. The
nurse should:
a. Consider this a normal finding.
b. Palpate this area for an underlying mass.
c. Reposition the hands, and attempt to percuss in this area again.
d. Consider this finding as abnormal, and refer the patient for additional treatment.
ANS: A
Percussion over relatively dense organs, such as the liver or spleen, will produce a dull
sound. The other responses are not correct.
10. The nurse is unable to identify any changes in sound when percussing over the
abdomen of an obese patient. What should the nurse do next?
a. Ask the patient to take deep breaths to relax the abdominal musculature.
b. Consider this finding as normal, and proceed with the abdominal assessment.
c. Increase the amount of strength used when attempting to percuss over the abdomen.
d. Decrease the amount of strength used when attempting to percuss over the abdomen.
ANS: C
The thickness of the persons body wall will be a factor. The nurse needs a stronger
percussion stroke for persons with obese or very muscular body walls. The force of
the blowdetermines the loudness of the note. The other actions are not correct.
11. The nurse hears bilateral loud, long, and low tones when percussing over the lungs of
a 4-year-old child. The nurse should:
Downloaded byAnna Maina(annamurugijoe@gmail.com)
lO M oA R cP S D| 1 2 2 6 3 4 2 3
a. Palpate over the area for increased pain and tenderness.
b. Ask the child to take shallow breaths, and percuss over the area again.
c. Immediately refer the child because of an increased amount of air in the lungs.
d. Consider this finding as normal for a child this age, and proceed with the examination.
ANS: D
Percussion notes that are loud in amplitude, low in pitch, of a booming quality, and long
in duration are normal over a childs lung.
12. A patient has suddenly developed shortness of breath and appears to be in
significant respiratory distress. After calling the physician and placing the patient on
oxygen, which of these actions is the best for the nurse to take when further assessing
the patient?
a. Count the patients respirations.
b. Bilaterally percuss the thorax, noting any differences in percussion tones.
c. Call for a chest x-ray study, and wait for the results before beginning an assessment.
d. Inspect the thorax for any new masses and bleeding associated with respirations.
ANS: B
Percussion is always available, portable, and offers instant feedback regarding changes in
underlying tissue density, which may yield clues of the patients physical status.
13. The nurse is teaching a class on basic assessment skills. Which of these statements
is true regarding the stethoscope and its use?
a. Slope of the earpieces should point posteriorly (toward the occiput).
b. Although the stethoscope does not magnify sound, it does block out extraneous room no
c. Fit and quality of the stethoscope are not as important as its ability to magnify sound.
d. Ideal tubing length should be 22 inches to dampen the distortion of sound.
ANS: B
The stethoscope does not magnify sound, but it does block out extraneous room
sounds. The slope of the earpieces should point forward toward the examiners nose.
Long tubing will distort sound. The fit and quality of the stethoscope are both
important.
14. The nurse is preparing to use a stethoscope for auscultation. Which statement
is true regarding the diaphragm of the stethoscope? The diaphragm:
a. Is used to listen for high-pitched sounds.
b. Is used to listen for low-pitched sounds.
c. Should be lightly held against the persons skin to block out low-pitched sounds.
d. Should be lightly held against the persons skin to listen for extra heart sounds and murm
Downloaded byAnna Maina(annamurugijoe@gmail.com)
lO M oA R cP S D| 1 2 2 6 3 4 2 3
ANS: A
The diaphragm of the stethoscope is best for listening to high-pitched sounds such as
breath, bowel, and normal heart sounds. It should be firmly held against the persons
skin, firmly enough to leave a ring. The bell of the stethoscope is best for soft, low-
pitched sounds such as extra heart sounds or murmurs.
15. Before auscultating the abdomen for the presence of bowel sounds on a patient, the
nurse should:
a. Warm the endpiece of the stethoscope by placing it in warm water.
Leave the gown on the patient to ensure that he or she does not get chilled during the
b. examination.
c. Ensure that the bell side of the stethoscope is turned to the on position.
d. Check the temperature of the room, and offer blankets to the patient if he or she feels col
ANS: D
The examination room should be warm. If the patient shivers, then the involuntary
muscle contractions can make it difficult to hear the underlying sounds. The end of the
stethoscope should be warmed between the examiners hands, not with water. The nurse
should never listen through a gown. The diaphragm of the stethoscope should be used to
auscultate for bowel sounds.
16. The nurse will use which technique of assessment to determine the presence of
crepitus, swelling, and pulsations?
a. Palpation
b. Inspection
c. Percussion
d. Auscultation
ANS: A
Palpation applies the sense of touch to assess texture, temperature, moisture, organ
location and size, as well as any swelling, vibration or pulsation, rigidity or spasticity,
crepitation, presence of lumps or masses, and the presence of tenderness or pain.
17. The nurse is preparing to use an otoscope for an examination. Which statement
is true regarding the otoscope? The otoscope:
a. Is often used to direct light onto the sinuses.
b. Uses a short, broad speculum to help visualize the ear.
c. Is used to examine the structures of the internal ear.
d. Directs light into the ear canal and onto the tympanic membrane.
ANS: D
Downloaded byAnna Maina(annamurugijoe@gmail.com)
lO M oA R cP S D| 1 2 2 6 3 4 2 3
The otoscope directs light into the ear canal and onto the tympanic membrane that
divides the external and middle ear. A short, broad speculum is used to visualize the
nares.
18. An examiner is using an ophthalmoscope to examine a patients eyes. The patient
has astigmatism and is nearsighted. The use of which of these techniques would
indicate that the examination is being correctly performed?
a. Using the large full circle of light when assessing pupils that are not dilated
b. Rotating the lens selector dial to the black numbers to compensate for astigmatism
c. Using the grid on the lens aperture dial to visualize the external structures of the eye
d. Rotating the lens selector dial to bring the object into focus
ANS: D
The ophthalmoscope is used to examine the internal eye structures. It can compensate
for nearsightedness or farsightedness, but it will not correct for astigmatism. The grid is
used to assess size and location of lesions on the fundus. The large full spot of light is
used to assess dilated pupils. Rotating the lens selector dial brings the object into focus.
19. The nurse is unable to palpate the right radial pulse on a patient. The best action
would be to:
a. Auscultate over the area with a fetoscope.
b. Use a goniometer to measure the pulsations.
c. Use a Doppler device to check for pulsations over the area.
d. Check for the presence of pulsations with a stethoscope.
ANS: C
Doppler devices are used to augment pulse or blood pressure measurements.
Goniometers measure joint range of motion. A fetoscope is used to auscultate fetal heart
tones. Stethoscopes are used to auscultate breath, bowel, and heart sounds.
20. The nurse is preparing to perform a physical assessment. The correct action by the
nurse is reflected by which statement? The nurse:
a. Performs the examination from the left side of the bed.
b. Examines tender or painful areas first to help relieve the patients anxiety.
c. Follows the same examination sequence, regardless of the patients age or condition.
d. Organizes the assessment to ensure that the patient does not change positions too often.
ANS: D
The steps of the assessment should be organized to ensure that the patient does not
change positions too often. The sequence of the steps of the assessment may differ,
depending on the age of the person and the examiners preference. Tender or painful
areas should be assessed last.
Downloaded byAnna Maina(annamurugijoe@gmail.com)
lO M oA R cP S D| 1 2 2 6 3 4 2 3
21. A man is at the clinic for a physical examination. He states that he is very anxious
about the physical examination. What steps can the nurse take to make him more
comfortable?
a. Appear unhurried and confident when examininghim.
b. Stay in the room when he undresses in case he needs assistance.
c. Ask him to change into an examining gown and to take off his undergarments.
Defer measuring vital signs until the end of the examination, which allows him time to b
d.
ANS:A
comfortable.
Anxiety can be reduced by an examiner who is confident, self-assured, considerate,
and unhurried. Familiar and relatively nonthreatening actions, such as measuring the
persons vital signs, will gradually accustom the person to the examination.
22. When performing a physical examination, safety must be considered to protect the
examiner and the patient against the spread of infection. Which of these statements
describes the most appropriate action the nurse should take when performing a
physicalexamination?
Washing ones hands after removing gloves is not necessary, as long as the gloves are sti
a. intact.
b. Hands are washed before and after everyphysical patient encounter.
Hands are washed before the examination of each body system to prevent the spread of
c. bacteria from one part of the body to another.
Gloves are worn throughout the entire examination to demonstrate to the patient concern
d.
ANS:B
regardingthe spread of infectious diseases.
The nurse should wash his or her hands before and after every physical patient
encounter; after contact with blood, body fluids, secretions, and excretions; after
contact with any equipment contaminated with body fluids; and after removing gloves.
Hands should be washed after gloves have been removed, even if the gloves appear to
be intact. Gloves should be worn when potential contact with any body fluids is present.
23. The nurse is examining a patients lower leg and notices a draining ulceration.
Which of these actions is most appropriate in this situation?
a. Washinghands, and contactingthe physician
b. Continuing to examine the ulceration, and then washing hands
c. Washing hands, putting on gloves, and continuing with the examination of the ulceration
Washinghands, proceeding with rest of the physical examination, and then continuing w
d. examination of the leg ulceration
Downloaded byAnna Maina(annamurugijoe@gmail.com)
lO M oA R cP S D| 1 2 2 6 3 4 2 3
ANS: C
The examiner should wear gloves when the potential contact with any body fluids is
present. In this situation, the nurse should wash his or her hands, put on gloves, and
continue examining the ulceration.
24. During the examination, offering some brief teaching about the patients body or
the examiners findings is often appropriate. Which one of these statements by the
nurse is most appropriate?
a. Your atrial dysrhythmias are under control.
b. Youhave pitting edema and mild varicosities.
c. Your pulse is 80 beats per minute, which is within the normal range.
d. Imusing my stethoscope to listen for any crackles, wheezes, or rubs.
ANS: C
The sharing of some information builds rapport, as long as the patient is able to
understand the terminology.
25. The nurse keeps in mind that the most important reason to share information and to
offer brief teaching while performing the physical examination is to help the:
a. Examiner feel more comfortable and to gain control of the situation.
b. Examiner to build rapport and to increase the patients confidence in him or her.
c. Patient understand his or her disease process and treatment modalities.
d. Patient identify questions about his or her disease and the potential areas of patient educ
ANS: B
Sharing information builds rapport and increases the patients confidence in the
examiner. It also gives the patient a little more control in a situation during which feeling
completely helpless is often present.
Downloaded byAnna Maina(annamurugijoe@gmail.com)
lO M oA R cP S D| 1 2 2 6 3 4 2 3
Chapter 5: Amenorrhea
1. If amenorrhea is reported by the female patient, the clinician should attribute this to until proven otherwise.
A. Pregnancy
B. Ovarian failure
C. Lack of progesterone
D. Menopause
2. A 17-year-old female patient complains of amenorrhea for the last 8 months, weight gain, excessive hair growth on the arms and
chest, and development of acne. Pelvic examination is normal. The clinician should recognize these are signs of:
A. Ovarian cancer
B. Endometriosis
C. Hormonal imbalance
D. Stress-induced anovulation
3.. A 55-year-old woman complains of amenorrhea and hot flashes for the last 3 months. Pregnancy test is negative. The clinician
should recognize that menopause is defined as the absence of menses for:
A. 3 months
B. 6 months
C. 1 year
D. 18 months
4. To confirm the diagnosis of amenorrhea due to menopause, blood should be drawn for hormones. Which are the changes that
occur with menopause?
A. FSH rises
B. LH decreases
C. LH rises
D. A & C
1. ANS: A
Last menstrual period is one of the most important questions to ask, particularly when prescribing medications, because many are
contraindicated in pregnancy. If menstrual cycles are not regular, pregnancy should be ruled out first, and then other diagnoses can
be considered. The menstrual history includes any episodes of amenorrhea, menorrhagia (excessive bleeding at the time of the
menstrual cycle), metrorrhagia (bleeding at irregular noncyclic intervals), dysmenorrhea, and postmenopausal bleeding.
Amenorrhea has many causes, including pregnancy; anorexia nervosa; excessive exercise; low body fat; and disorders or tumors of
the hypothalamus, pituitary gland, ovary, uterus, and thyroid gland.
2.. ANS: C
Anovulation may present with amenorrhea but also may present with dysfunctional uterine bleeding, polymenorrhea, or
menorrhagia. The symptoms vary with the cause. Overweight may be seen with several of the causes, including hypothyroidism,
polycystic ovarian syndrome, and pituitary and adrenal dysfunction. Underweight is seen in anorexia nervosa, excessive exercise,
hyperthyroidism, or stress-induced anovulation. Hirsutism, acne, and other skin changes can be seen with imbalances in LH, FSH,
and androgens, as seen in polycystic ovary disease. Delayed puberty or regression of sexual characteristics is seen in hypopituitarism;
galactorrhea can be the presenting symptom in pituitary tumors.
PTS: 1
3.. ANS: B
Menopause is the absence of menses for at least 6 months. Age of menopause varies greatly, although age 50 to 55 is the typical
range of onset. Although the absence of menses in a woman around the age of 50 years is diagnostic for menopause, measurement
of FSH, LH, and estradiol levels are helpful in confirming the diagnosis.
PTS: 1
4.. ANS: D
In menopause, FSH rises first and then LH rises, both greater than 100 mU/mL. A fall in estradiol is the last hormonal change that
occurs with the decline of ovarian function. An estradiol level of less than 30 pg/mL indicates loss of ovarian function.
Downloaded byAnna Maina(annamurugijoe@gmail.com)
lO M oA R cP S D| 1 2 2 6 3 4 2 3
Chapter 6. Breasts lumps and nipple discharge
Multiple Choice
Identify the choice that best completes the statement or answers the question.
1. Your patient has a rubbery, firm, mobile breast mass. In order to completely exclude the possibility of cancer, the triple
test is necessary, which includes:
A. CT scan
B. MRI
C. Biopsy
D. All of the above
2. A23-year-old woman is concerned about a mass she found on palpation. Upon examination of the patient, the mass
feels cystic, round, and mobile with discrete borders. The diagnostic study that is recommended is:
A. CT scan
B. Mammogram
C. Biopsy
D. Ultrasound
3. Which of the following variables is not a component of the Gail Model?
A. Age at menarche
B. Number of breast biopsies
C. Age at first live birth
D. Number of live births
4. A62-year-old female presents with a singular, hard, 1 cm, non-tender, non-mobile mass in the right breast. There are
no nipple or skin changes, however, you palpate an enlarged right-sided supraclavicular lymph node. The clinician should recognize
these are signs of:
A. Fibroadenoma
B. Breast cyst
C. Malignancy
D. Paget’s disease
5. A45-year-old female patient presents in the emergencydepartment due to multiple chest injuries as a result of a motor
vehicle accident. There is a palpable, tender, irregular 3 cm soft mass located in the left breast, with erythema and swelling of the
breast. Which of the following is indicated?
A. Compression wrap for chest and breast trauma
B. Mammogram
C. Re-evaluation following complete resolution of obvious injuries
D. Ultrasound
6. Your patient is a 46-year-old woman suffering from psychosis complains of milkydischarge from both her breasts.
Upon examination, there are no significant findings and the patient is not pregnant, breastfeeding, or in the postpartum period. You
should:
A. Order CT scan of head to rule out pituitary tumor
B. Obtain a complete list of the patient’s medications
C. Have the patient keep a menstrual cycle diary
D. Refer the patient for mammogram
7.. Your patient is a 36-year-old woman who complains of milky discharge from both her breasts, episodes of headache, and
menstrual irregularity. The patient is on no medications. Upon examination, there are no significant findings, and the patient is not
pregnant, breastfeeding, or in the postpartum period. It is important to:
A. Order prolactin level
B. Obtain an MRI of head
C. Have the patient keep a menstrual cycle diary
D. Refer the patient for mammogram
Downloaded byAnna Maina(annamurugijoe@gmail.com)
lO M oA R cP S D| 1 2 2 6 3 4 2 3
8. Bloodybreast discharge is associated with which of the following?
A. Bleeding disorder such as von Willebrand disease
B. Malignancy
C. Excessive non-steroidal anti-inflammatories
D. A and C
9. Your patient is a 64-year-old female who presents with complaints of a red, scaly rash on her nipple and areola of one
breast. It has been present for a few months. There is no mass or nipple discharge. You should be suspicious of:
A. Contact dermatitis
B. Mastitis
C. Paget’s disease
D. Eczema
10. A12-year-old boyis brought into the clinic byhis mother. The teen boyis embarrassed because he has developed
breast tissue and would like to know if there is any treatment. The physical examination reveals early stages of puberty, otherwise it is
normal. This presentation is:
A. Associated with a high rate of malignancy
B. Most often due to altered hormonal levels in puberty
C. Commonlyindicative of pituitary tumor triggered by puberty
D. Too dense for a mammogram and requires biopsy
11 A condition that increases risk of breast cancer in a male is:
A. Pseudogynecomastia
B. BRCA1 and 2
C. Klinefelter’s syndrome
D. B and C
Chapter 6. Breasts lumps and nipple discharge
Answer Section
MULTIPLE CHOICE
PTS: 1
1. ANS: C
Palpation and mammography, alone or together, are inadequate to definitively identify the cause of a breast mass and to rule out
malignancy. The “triple test” recommended for evaluation of a breast mass involves clinical examination, either ultrasound or
mammogram, and aspiration and/or biopsy. The determination of whether an ultrasound or mammography is recommended is
based on age and other situations.
PTS: 1
2. ANS: D
The American College of Radiology (2012) recommends that women under 40 years of age should be assessed through ultrasound,
and women who are older than 40 years should be assessed with a mammogram. Younger women have denser breasts, and ultrasound
is, therefore, often more useful than a mammogram. Ultrasounds are helpful in determining whether or not a mass that
feels potentially “cystic” is fluid filled or solid. It is crucial that the woman understand that the imaging procedure is a screening tool
and is never diagnostic regarding the existence or absence of a malignancy.
PTS: 1
3. ANS: D
The Gail model is a clinical prediction rule used to estimate a patient’s risk for breast cancer. The model identifies the relative risk
associated with three factors obtained through history: age at menarche, number of previous breast biopsies, and the age at first live
birth.
PTS: 1
4. ANS: C
The typical malignant breast mass is solitary, non-tender, hard, immobile or fixed, and poorly defined. It may be accompanied by
nipple erosion or other inflammatory skin changes, as seen in Paget’s disease; nipple discharge; skin thickening or dimpling; retraction;
and palpable axillary nodes. Although most malignant masses are painless, associated discomfort does not exclude the potential for
breast cancer.
Downloaded byAnna Maina(annamurugijoe@gmail.com)
lO M oA R cP S D| 1 2 2 6 3 4 2 3
PTS: 1
5. ANS: B
Trauma to the anterior chest area may result in a palpable breast mass. An automobile accident with injury from contact with the seat
belt, air bag, steering wheel, or dashboard is a common source of breast trauma. When a palpable mass results from chest trauma, it
typically represents either a hematoma or area of secondary fat necrosis. Even when a mass is identified subsequent to direct trauma,
the provider must remain suspicious for the possibility of malignancy that preexisted but was undetected before the accident.
PTS: 1
6. ANS: B
Galactorrhea is characterized by bilateral and milky discharge from multiple ducts in a woman who is neither pregnant nor lactating.
Causes of galactorrhea include a variety of drugs as well as an elevated prolactin level associated with pituitary tumor or
hyperthyroidism. The drugs associated with galactorrhea include antidepressants (amitriptyline, imipramine), psychoactives
(haloperidol, thioridazine), hormones (estrogens, progestogens), antiepileptics (valproic acid), and antihypertensives (verapamil).
This list is not exhaustive.
PTS: 1
7. ANS: A
Galactorrhea can be a symptom of endocrine disorder, particularly pituitary dysfunction. In prolactinoma of the pituitary, a
patient complains of galactorrhea, headaches, vision change, relative infertility, and amenorrhea. Laboratory studies should
include a pregnancy test, prolactin level, and thyroid functions.
PTS: 1
8. ANS: B
Bloody discharge is often associated with malignancy but can stem from other conditions. If blood is not evident, the discharge
should be tested for blood, using guaiac process.
PTS: 1
9 ANS: C
In Paget’s disease of the breast, the patient may describe the persistence of skin changes for several months. The typical
presentation involves skin changes of the nipple and/or areola, with the nipple being involved first. The condition does not always
involve a palpable mass or nodule. The skin changes of the nipple and areola range from scaling redness to various degrees of
ulceration.
PTS: 1
10. ANS: B
Gynecomastia most often occurs during infancy, puberty, and senescence. It is caused by an altered balance between estradiol and
testosterone levels. Although it can be an indication of primary hypogonadism, hyperthyroidism, cirrhosis, or renal disease, the
majority of the cases are specific to hormonal changes of puberty, are drug induced, or are idiopathic. With the presentation of
breast enlargement in a male, malignancy must always be considered.
PTS: 1
11 ANS: D
It is anticipated that over 2,000 new cases of male breast cancer will be diagnosed in 2013, with over 400 related deaths (ACS 2013).
Men develop the same types of breast cancer as women. Therefore, it is important to include malignancy in the differential diagnosis
when a man complains of breast enlargement. Factors that increase the risk of breast cancer in men include a previous history of
breast or testicular disease and Klinefelter’s syndrome. A history of gynecomastia is not associated with an increased risk for breast
cancer.
Downloaded byAnna Maina(annamurugijoe@gmail.com)
lO M oA R cP S D| 1 2 2 6 3 4 2 3
Chapter 7. Breast Pain
Multiple Choice
Identify the choice that best completes the statement or answers the question.
1. When examining the breasts of a 45-year-old female patient, you note nodularity throughout both breasts. There is no
dominant mass, tenderness, nipple change, or skin change. Which of the following condition is most likely?
A. Multiple fibroadenomas
B. Fibrocystic breasts
C. Mastalgia
D. Mastitis
2. A 23-year-old female presents with episodicbilateral breast tenderness. Upon palpation, there are multiple areas of
nodularitywith no dominant breast mass and no tenderness, nipple, or skin changes. It is best torecommend:
A. Ultrasound of the breasts
B. Mammogram of the breasts
C. Fine needle aspiration of one nodule within a breast
D. Diaryof menstrual cycle and breast symptoms
3.. A 34-year-old female who gave birth 2 weeks ago developed fever and pain in the right breast while nursing her baby. The
patient has tenderness, eythema, and swelling of the nipple on the right breast. Which of the following diagnostic studies is
indicated?
A. White blood cell (WBC) count
B. Breast milk culture
C. Ultrasound
D. Fine needle biopsy
4.. A 52-year-old female complains of discharge from one of her breasts. There is no pain, no mass, and no skin changes. The
physical examination is normal. When putting pressure on the affected breast, the nipple expresses a small amount of thick, white
discharge. These findings are consistent with:
A. Duct ectasia
B. Mastitis
C. Fibroadenoma
D. Fibrocystic breasts
Chapter 7. Breast Pain
Answer Section
MULTIPLE CHOICE
1. ANS: A
Breast tissue is normally glandular and may have a rather nodular consistency. The degree of nodularity tends to fluctuate through
the menstrual cycle in premenopausal women. A dominant breast mass is a mass that persists throughout a woman’s hormonal cycles,
is larger and firmer than any other irregularities, and differs from rest of the breast tissue. A fibroadenoma is a soft discrete breast
mass that is moveable, non-tender, and benign. Mastalgia is a condition of pain in the breast. Mastitis is an infection that causes
erythema, swelling, and discharge from the breast.
PTS: 1
2.. ANS: D
Women who experience cyclic mastalgia usually have onset as a teen or young adult. It is important to determine menstrual and
reproductive history and to identify all pharmacologic agents taken. A complete breast examination should be performed. The pain
associated with hormonal fluctuation most commonly occurs during the second half of the woman’s cycle. The variability of the signs
and symptoms is identified with a symptom diary and record of the menstrual cycle. The pain is typically poorly localized, bilateral,
and nonspecific. It may be accompanied by a sense of breast fullness. The examination may identify the multiple, bilateral nodularities
associated with fibroadenomas or fibrocystic changes.
PTS: 1
Downloaded byAnna Maina(annamurugijoe@gmail.com)
lO M oA R cP S D| 1 2 2 6 3 4 2 3
3. ANS: A
For mastitis, white blood count should be obtained and is usually elevated. Even though the breast milk can be cultured, this is not
generally recommended. If the presentation is atypical—that is, the patient is not lactating—and there are no associated systemic signs
or symptoms, a consultation should be obtained and mammography ordered to determine the definitive diagnosis and rule out
malignancy.
PTS: 1
4.. ANS: A
Discharge from one breast is commonly benign and frequently caused by duct ectasia or an intraductal papilloma. Duct ectasia results
in dilation of one major breast duct and causes approximately one-third of the cases of pathological discharge. Papillomas are
responsible for 44% of pathologic discharge. Because the discharge associated with ductal ectasia is often stagnant, the discharge is
cheesy in appearance. It is often associated with noncyclic breast discomfort and a subareolar lump at the site of the dilated duct.
Because one major duct is involved, the discharge comes also from one duct or nipple area.
PTS: 1
Downloaded byAnna Maina(annamurugijoe@gmail.com)
lO M oA R cP S D| 1 2 2 6 3 4 2 3
Chapter 8: Chest Pain
1.
A.
Cardiac chest pain is most often described as:
Stabbing, piercing pain
B.
Pain with inhalation
C.
Crushing, squeezing pain
D.
Burning, gnawing pain
2.
A.
The pain associated with pericarditis is .
Crushing and squeezing
B.
Constant
C.
Worse with inspiration
D.
Onlypresent with fever
3. Pain associated with a dissecting thoracic aortic aneurysm is commonlydescribed as:
A. Retrosternal crushing and squeezing
B. Chest stabbing and sharp
C. Ripping and tearing in the chest or thoracic back
D. Worse with inspiration
4. The pain of can frequentlybe mistaken for cardiac chest pain.
A. Gastroesophageal reflux disease (GERD)
B. Peptic ulcer disease (PUD)
C. Cholecystitis
D. All of the above
5. The pain of pancreatitis is described as:
A. Abdominal sharp and piercing pain in the left upper quadrant
B. Dull and cramping pain in the right upper quadrant
C. Severe, epigastric pain radiating straight into the back
D. Sharp pain radiating to the shoulder
6. The pain of costochondritis typically .
A. Mimics cardiac crushing and squeezing pain
B. Worsens with movement and full inspiration
C. Radiates from epigastrium into the back
D. Is a tearing and ripping pain
7. Patients with anxiety frequently complain of:
A. Chest pain
B. Dizziness
C. Shortness of breath
D. A and B
8. . During a cardiac assessment on a 38-year-old patient in the hospital for chest pain, the nurse finds the following: jugular vein
pulsations 4 cm above the sternal angle when the patient is elevated at 45 degrees, blood pressure 98/60 mm Hg, heart rate 130 beats
per minute, ankle edema, difficultybreathing when supine, and an S3 on auscultation. Which of these conditions best
explains the cause of these findings?
a. Fluid overload
Downloaded byAnna Maina(annamurugijoe@gmail.com)
lO M oA R cP S D| 1 2 2 6 3 4 2 3
b. Atrial septal defect
c. MI
d. Heart failure
ANS: D
1. ANS: C
Typical characteristics that indicate acute coronary syndrome (ACS) include crushing, squeezing substernal chest pain with radiation
to the neck or left arm, a score of greater than 7 on the pain scale, an association with exertion or stress with relief on rest, a duration
of minutes, and associated symptoms of nausea, diaphoresis, weakness, or shortness of breath.
2. ANS: C
Unlike the symptoms associated with ACS, the pain associated with pericarditis is sharp and stabbing; it may worsen with
inspiration or when lying flat or leaning forward. Associated symptoms may include shortness of breath, fever, chills, and
malaise.
PTS: 1
3. ANS: C
Aortic aneurysms are often asymptomatic. However, in a dissecting aortic aneurysm , symptoms are often described as tearing or
ripping in the chest, back, or abdomen.
PTS: 1
4. ANS: D
It is often difficult to differentiate the symptoms of gastroesophageal reflux disease (GERD) or peptic ulcer disease (PUD) from
cardiac symptoms. A thorough history and diagnostic tests are necessary. Patients with a history of GERD or PUD should still be
worked up for a cardiac etiology, particularly if the characteristics of the symptoms or the history have changed to raise the index of
suspicion for cardiac disease. The pain of cholecystitis, also sometimes mistaken for cardiac pain, typically presents with right upper
quadrant pain with radiation to the thoracic region of the back.
PTS: 1
5. ANS: C
The pain of pancreatitis is severe, steady, and “boring”—radiating from the epigastric region through to the back. It is often
accompanied by nausea and vomiting, tachycardia, hypotension, and diaphoresis. These symptoms are also seen in MI; however, the
exquisite abdominal tenderness present in pancreatitis assists in differentiating it from cardiac pain.
PTS: 1
6. ANS: B
Costochondritis, which is inflammatory pain of the chest wall, can often be differentiated from cardiac pain through history. A
history of injury, heavy lifting, contact sports, excessive coughing, or late-stage pregnancy (which stretches the intercostal muscles)
leads the examiner to consider chest wall pain. This often occurs in a younger population with no cardiac risk factors. One of the
most helpful differentiating symptoms is that the pain is increased with movement, cough, or, in some cases, respiration.
7. ANS: D
More than 90% of patients with anxiety present primarily with somatic complaints in primary care and emergency department
settings (Stern & Herman, 2003). Patients may initially complain of only somatic symptoms before they are ultimately diagnosed with
a primary anxiety disorder (DeVane et al., 2005). Patients with anxiety often present with the following symptoms: chest pain (with
negative angiogram), irritable bowel, unexplained dizziness, migraine headache, and chronic fatigue.
8. Ans. D.Heart failure causes decreased cardiac output when the heart fails as a pump and the circulation becomes backed up and
congested. Signs and symptoms include dyspnea, orthopnea, paroxysmal nocturnal dyspnea, decreased blood pressure,
Downloaded byAnna Maina(annamurugijoe@gmail.com)
lO M oA R cP S D| 1 2 2 6 3 4 2 3
dependent and pitting edema; anxiety; confusion; jugular vein distention; and fatigue. The S3 is associated with heart failure and is
always abnormal after 35 years of age. The S3 maybe the earliest sign of heart failure.
Downloaded byAnna Maina(annamurugijoe@gmail.com)
lO M oA R cP S D| 1 2 2 6 3 4 2 3
Chapter 9. Confusion in older adults
Multiple Choice
Identify the choice that best completes the statement or answers the question.
1. An 85-year-old female patient arrives by ambulance to the emergency department accompanied by
her husband. He reports that his wife had been ill with pneumonia and, 2 days ago, went to the
family physician who prescribed azithromycin twice a day. The husband reports that he is making
sure she gets the medicine. His wife has been staying in bed and resting. She awoke from sleep last
night and was extremely agitated, left the house, and was walking outside. She did not recognize her
husband and wanted to call the police. Which of the following is an appropriate question for the
history?
A. Does your wife have dementia or frequent episodes of confusion?
B. Has your wife been running a fever?
C. Is your wife allergic to any medication?
D. Has your wife ever had a mental status exam?
PTS: 1
ANS: A
Delirium can be observed in both elderly and younger patients and is generally defined as an acute
confusional state, affecting all aspects of cognition and mentation. The signs and symptoms of
delirium generally have a more acute or rapidly progressive onset as opposed to the slow, gradual
decline noted in the organic dementias. The acute mental status change is often associated with
other signs or symptoms—such as hallucinations, illusions, incoherent speech, and constant
aimless activity—that help to narrow the differential diagnosis. Electrolyte disturbances, infection,
and polypharmacy are frequent causes of delirium in the elderly.
2. An older patient has been admitted to the intensive care unit (ICU) after falling at home.
Within 8 hours, his condition has stabilized and he is transferred to a medical unit. The family is
wondering whether he will be able to go back home. Which assessment instrument is most
appropriate for the nurse to choose at this time?
a. Lawton IADL instrument
b. Hospital Admission Risk Profile (HARP)
c. Mini-Cog
d. NEECHAM Confusion Scale
ANS: B
Hospital-acquired functional decline may occur within 2 days of a hospital admission. The HARP
helps identify older adults who are at greatest risk of losing their ability to perform ADLs or
mobility at this critical time. The Lawton IADL measures instrumental activities of daily living,
which may be difficult to observe in the hospital setting. The Mini-Cog is an assessment of mental
status. The NEECHAM Confusion Scale is used to assess for delirium.
Downloaded byAnna Maina(annamurugijoe@gmail.com)
lO M oA R cP S D| 1 2 2 6 3 4 2 3
3. During a functional assessment of an older persons home environment, which statement or
question by the nurse is most appropriate regarding common environmental hazards?
a. These low toilet seats are safe because they are nearer to the ground in case of falls.
b. Do you have a relative or friend who can help to install grab bars in your shower?
c. These small rugs are ideal for preventing you from slipping on the hard floor.
d. It would be safer to keep the lighting low in this room to avoid glare in your eyes.
ANS: B
Environmental hazards within the home can be a potential constraint on the older persons day-to-
day functioning. Common environmental hazards, including inadequate lighting, loose throw rugs,
curled carpet edges, obstructed hallways, cords in walkways, lack of grab bars in tub and shower,
and low and loose toilet seats, are hazards that could lead to an increased risk of falls and fractures.
Environmental modifications can promote mobility and reduce the likelihood of the older adult
falling.
4. When assessing aging adults, the nurse knows that one of the first things that should be
assessed before making judgments about their mental status is:
a. Presence of phobias
b. General intelligence
c. Presence of irrational thinking patterns
d. Sensory-perceptive abilities
ANS: D
Age-related changes in sensoryperception can affect mental status. For example, vision loss (as
detailed in Chapter 14) may result in apathy, social isolation, and depression. Hearing changes are
common in older adults, which produces frustration, suspicion, and social isolation and makes the
person appear confused.
5. During the taking of the health history of a 78-year-old man, his wife states that he occasionally
has problems with short-term memory loss and confusion: He cant even remember how to button
his shirt. When assessing his sensory system, which action by the nurse is most appropriate?
a.The nurse would not test the sensory system as part of the examination because the results
would not be valid.
b.The nurse would perform the tests, knowing that mental status does not affect sensory ability.
c.The nurse would proceed with an explanation of each test, making certain that the wife
understands.
d.Before testing, the nurse would assess the patients mental status and ability to follow
directions.
Test bank for advanced assessment interpreting findings and formulating differential diagnoses 4th edition - Copy.pdf
Test bank for advanced assessment interpreting findings and formulating differential diagnoses 4th edition - Copy.pdf
Test bank for advanced assessment interpreting findings and formulating differential diagnoses 4th edition - Copy.pdf
Test bank for advanced assessment interpreting findings and formulating differential diagnoses 4th edition - Copy.pdf
Test bank for advanced assessment interpreting findings and formulating differential diagnoses 4th edition - Copy.pdf
Test bank for advanced assessment interpreting findings and formulating differential diagnoses 4th edition - Copy.pdf
Test bank for advanced assessment interpreting findings and formulating differential diagnoses 4th edition - Copy.pdf
Test bank for advanced assessment interpreting findings and formulating differential diagnoses 4th edition - Copy.pdf
Test bank for advanced assessment interpreting findings and formulating differential diagnoses 4th edition - Copy.pdf
Test bank for advanced assessment interpreting findings and formulating differential diagnoses 4th edition - Copy.pdf

More Related Content

Similar to Test bank for advanced assessment interpreting findings and formulating differential diagnoses 4th edition - Copy.pdf

Assessment of the Genitalia and RectumStudent Na.docx
Assessment of the Genitalia and RectumStudent Na.docxAssessment of the Genitalia and RectumStudent Na.docx
Assessment of the Genitalia and RectumStudent Na.docx
galerussel59292
 
Assessment of the Genitalia and RectumStudent Na.docx
Assessment of the Genitalia and RectumStudent Na.docxAssessment of the Genitalia and RectumStudent Na.docx
Assessment of the Genitalia and RectumStudent Na.docx
festockton
 
Introduction to Clinical Epidemiology (401173) FINAL ASSIGNMENT
Introduction to Clinical Epidemiology (401173) FINAL ASSIGNMENTIntroduction to Clinical Epidemiology (401173) FINAL ASSIGNMENT
Introduction to Clinical Epidemiology (401173) FINAL ASSIGNMENT
hildredzr1di
 
Name Professor Course Date Sexual Harassment .docx
Name  Professor  Course  Date   Sexual Harassment .docxName  Professor  Course  Date   Sexual Harassment .docx
Name Professor Course Date Sexual Harassment .docx
roushhsiu
 
see attachments I have complete a portion of the assignment but need
see attachments I have complete a portion of the assignment but needsee attachments I have complete a portion of the assignment but need
see attachments I have complete a portion of the assignment but need
PazSilviapm
 
Nursing diagnosis for nurses
Nursing diagnosis for nurses Nursing diagnosis for nurses
Nursing diagnosis for nurses
Taghreed Hawsawi
 
C H A P T E R 1 Clinical rea
C H A P T E R  1  Clinical reaC H A P T E R  1  Clinical rea
C H A P T E R 1 Clinical rea
TawnaDelatorrejs
 
Eblm pres final
Eblm pres finalEblm pres final
Eblm pres final
prasath172
 
The goal of patient interviews is to develop a therapeutic relations.docx
The goal of patient interviews is to develop a therapeutic relations.docxThe goal of patient interviews is to develop a therapeutic relations.docx
The goal of patient interviews is to develop a therapeutic relations.docx
rtodd194
 

Similar to Test bank for advanced assessment interpreting findings and formulating differential diagnoses 4th edition - Copy.pdf (20)

Assessment of the Genitalia and RectumStudent Na.docx
Assessment of the Genitalia and RectumStudent Na.docxAssessment of the Genitalia and RectumStudent Na.docx
Assessment of the Genitalia and RectumStudent Na.docx
 
Assessment of the Genitalia and RectumStudent Na.docx
Assessment of the Genitalia and RectumStudent Na.docxAssessment of the Genitalia and RectumStudent Na.docx
Assessment of the Genitalia and RectumStudent Na.docx
 
Introduction to Clinical Epidemiology (401173) FINAL ASSIGNMENT
Introduction to Clinical Epidemiology (401173) FINAL ASSIGNMENTIntroduction to Clinical Epidemiology (401173) FINAL ASSIGNMENT
Introduction to Clinical Epidemiology (401173) FINAL ASSIGNMENT
 
Case Report Writing
Case Report WritingCase Report Writing
Case Report Writing
 
EBM_2023.pptx
EBM_2023.pptxEBM_2023.pptx
EBM_2023.pptx
 
Fall Risk Case Study # 1
Fall Risk Case Study # 1Fall Risk Case Study # 1
Fall Risk Case Study # 1
 
Evidence Based Dentistry.pptx
Evidence Based Dentistry.pptxEvidence Based Dentistry.pptx
Evidence Based Dentistry.pptx
 
Name Professor Course Date Sexual Harassment .docx
Name  Professor  Course  Date   Sexual Harassment .docxName  Professor  Course  Date   Sexual Harassment .docx
Name Professor Course Date Sexual Harassment .docx
 
see attachments I have complete a portion of the assignment but need
see attachments I have complete a portion of the assignment but needsee attachments I have complete a portion of the assignment but need
see attachments I have complete a portion of the assignment but need
 
Personalized Medicine Overview
Personalized Medicine OverviewPersonalized Medicine Overview
Personalized Medicine Overview
 
Nursing diagnosis for nurses
Nursing diagnosis for nurses Nursing diagnosis for nurses
Nursing diagnosis for nurses
 
Introduction to Evidence Based Medicine (EBM)
Introduction to Evidence Based Medicine (EBM)Introduction to Evidence Based Medicine (EBM)
Introduction to Evidence Based Medicine (EBM)
 
Published Research, Flawed, Misleading, Nefarious - Use of Reporting Guidelin...
Published Research, Flawed, Misleading, Nefarious - Use of Reporting Guidelin...Published Research, Flawed, Misleading, Nefarious - Use of Reporting Guidelin...
Published Research, Flawed, Misleading, Nefarious - Use of Reporting Guidelin...
 
C H A P T E R 1 Clinical rea
C H A P T E R  1  Clinical reaC H A P T E R  1  Clinical rea
C H A P T E R 1 Clinical rea
 
Research.docx
Research.docxResearch.docx
Research.docx
 
Eblm pres final
Eblm pres finalEblm pres final
Eblm pres final
 
Brief introduction to different types of studies (Pyramid of Evidence)
Brief introduction to different types of studies (Pyramid of Evidence)Brief introduction to different types of studies (Pyramid of Evidence)
Brief introduction to different types of studies (Pyramid of Evidence)
 
Ebm Nahid Sherbini
Ebm Nahid SherbiniEbm Nahid Sherbini
Ebm Nahid Sherbini
 
23205042
2320504223205042
23205042
 
The goal of patient interviews is to develop a therapeutic relations.docx
The goal of patient interviews is to develop a therapeutic relations.docxThe goal of patient interviews is to develop a therapeutic relations.docx
The goal of patient interviews is to develop a therapeutic relations.docx
 

More from robinsonayot

More from robinsonayot (20)

Test bank for discovering the life span 4th edition robert s feldman (1).pdf
Test bank for discovering the life span 4th edition robert s feldman (1).pdfTest bank for discovering the life span 4th edition robert s feldman (1).pdf
Test bank for discovering the life span 4th edition robert s feldman (1).pdf
 
Test bank for discovering the life span 4th edition robert s feldman.pdf
Test bank for discovering the life span 4th edition robert s feldman.pdfTest bank for discovering the life span 4th edition robert s feldman.pdf
Test bank for discovering the life span 4th edition robert s feldman.pdf
 
TEST BANK FOR ESSENTIALS OF NURSING LEADERSHIP AND MANAGEMENT, 7TH EDITION.pdf
TEST BANK FOR ESSENTIALS OF NURSING LEADERSHIP AND MANAGEMENT, 7TH EDITION.pdfTEST BANK FOR ESSENTIALS OF NURSING LEADERSHIP AND MANAGEMENT, 7TH EDITION.pdf
TEST BANK FOR ESSENTIALS OF NURSING LEADERSHIP AND MANAGEMENT, 7TH EDITION.pdf
 
Test bank for focus on nursing pharmacology 7th edition by amy m karch.pdf
Test bank for focus on nursing pharmacology 7th edition by amy m karch.pdfTest bank for focus on nursing pharmacology 7th edition by amy m karch.pdf
Test bank for focus on nursing pharmacology 7th edition by amy m karch.pdf
 
Test bank for davis advantage for pathophysiology 2nd edition by caprio.pdf
Test bank for davis advantage for pathophysiology 2nd edition by caprio.pdfTest bank for davis advantage for pathophysiology 2nd edition by caprio.pdf
Test bank for davis advantage for pathophysiology 2nd edition by caprio.pdf
 
Test bank for current medical diagnosis and treatment 2023 2024 62nd edition ...
Test bank for current medical diagnosis and treatment 2023 2024 62nd edition ...Test bank for current medical diagnosis and treatment 2023 2024 62nd edition ...
Test bank for current medical diagnosis and treatment 2023 2024 62nd edition ...
 
Test bank for current diagnosis and treatment pediatrics twenty fourth editio...
Test bank for current diagnosis and treatment pediatrics twenty fourth editio...Test bank for current diagnosis and treatment pediatrics twenty fourth editio...
Test bank for current diagnosis and treatment pediatrics twenty fourth editio...
 
Test bank for critical care nursing a holistic approach 11th edition morton f...
Test bank for critical care nursing a holistic approach 11th edition morton f...Test bank for critical care nursing a holistic approach 11th edition morton f...
Test bank for critical care nursing a holistic approach 11th edition morton f...
 
Test bank for consumer behaviour buying having and being eighth canadian edit...
Test bank for consumer behaviour buying having and being eighth canadian edit...Test bank for consumer behaviour buying having and being eighth canadian edit...
Test bank for consumer behaviour buying having and being eighth canadian edit...
 
Test Bank For Comprehensive Radiographic Pathology 7th Edition By Eisenberg.pdf
Test Bank For Comprehensive Radiographic Pathology 7th Edition By Eisenberg.pdfTest Bank For Comprehensive Radiographic Pathology 7th Edition By Eisenberg.pdf
Test Bank For Comprehensive Radiographic Pathology 7th Edition By Eisenberg.pdf
 
Test bank for community public health nursing evidence for practice 4TH editi...
Test bank for community public health nursing evidence for practice 4TH editi...Test bank for community public health nursing evidence for practice 4TH editi...
Test bank for community public health nursing evidence for practice 4TH editi...
 
Test bank for clinical procedures for medical assistants 10th edition by bone...
Test bank for clinical procedures for medical assistants 10th edition by bone...Test bank for clinical procedures for medical assistants 10th edition by bone...
Test bank for clinical procedures for medical assistants 10th edition by bone...
 
Test bank for community health nursing a canadian perspective 5th edition by ...
Test bank for community health nursing a canadian perspective 5th edition by ...Test bank for community health nursing a canadian perspective 5th edition by ...
Test bank for community health nursing a canadian perspective 5th edition by ...
 
Test bank for beckmann and ling s obstetrics and gynecology 8th edition by ro...
Test bank for beckmann and ling s obstetrics and gynecology 8th edition by ro...Test bank for beckmann and ling s obstetrics and gynecology 8th edition by ro...
Test bank for beckmann and ling s obstetrics and gynecology 8th edition by ro...
 
Test Bank for Anatomy of Oriented Structure 8th edition.pdf
Test Bank for Anatomy of Oriented Structure 8th edition.pdfTest Bank for Anatomy of Oriented Structure 8th edition.pdf
Test Bank for Anatomy of Oriented Structure 8th edition.pdf
 
Test bank for advanced assessment interpreting findings and formulating diffe...
Test bank for advanced assessment interpreting findings and formulating diffe...Test bank for advanced assessment interpreting findings and formulating diffe...
Test bank for advanced assessment interpreting findings and formulating diffe...
 
TEST BANK Essentials of dental radiography 9th edition by Evelyn Thomson, Orl...
TEST BANK Essentials of dental radiography 9th edition by Evelyn Thomson, Orl...TEST BANK Essentials of dental radiography 9th edition by Evelyn Thomson, Orl...
TEST BANK Essentials of dental radiography 9th edition by Evelyn Thomson, Orl...
 
Test bank calculating drug dosages a patient safe approach to nursing and mat...
Test bank calculating drug dosages a patient safe approach to nursing and mat...Test bank calculating drug dosages a patient safe approach to nursing and mat...
Test bank calculating drug dosages a patient safe approach to nursing and mat...
 
Test bank advanced health assessment and differential diagnosis essentials fo...
Test bank advanced health assessment and differential diagnosis essentials fo...Test bank advanced health assessment and differential diagnosis essentials fo...
Test bank advanced health assessment and differential diagnosis essentials fo...
 
TEST BANK For Family Practice Guidelines, 5th Edition by Jill C. Cash; Cheryl...
TEST BANK For Family Practice Guidelines, 5th Edition by Jill C. Cash; Cheryl...TEST BANK For Family Practice Guidelines, 5th Edition by Jill C. Cash; Cheryl...
TEST BANK For Family Practice Guidelines, 5th Edition by Jill C. Cash; Cheryl...
 

Recently uploaded

Zepto Case study(On Track to Profitability).pptx
Zepto Case study(On Track to Profitability).pptxZepto Case study(On Track to Profitability).pptx
Zepto Case study(On Track to Profitability).pptx
aryan963438
 
State Space Tutorial.pptxjjjjjjjjjjjjjjj
State Space Tutorial.pptxjjjjjjjjjjjjjjjState Space Tutorial.pptxjjjjjjjjjjjjjjj
State Space Tutorial.pptxjjjjjjjjjjjjjjj
joshuaclack73
 
Chapter Three Interest rates in the Financial System.ppt
Chapter Three Interest rates in the Financial System.pptChapter Three Interest rates in the Financial System.ppt
Chapter Three Interest rates in the Financial System.ppt
Kalkaye
 
GROUP 6 DUBAI.pptx basta amoa na dira dapita
GROUP 6 DUBAI.pptx basta amoa na dira dapitaGROUP 6 DUBAI.pptx basta amoa na dira dapita
GROUP 6 DUBAI.pptx basta amoa na dira dapita
JohnThomas845833
 

Recently uploaded (20)

Indirect tax .pptx Supply under GST, Charges of GST
Indirect tax .pptx  Supply under GST, Charges of GSTIndirect tax .pptx  Supply under GST, Charges of GST
Indirect tax .pptx Supply under GST, Charges of GST
 
how do i sell pi coins in Pakistan at the best rate.
how do i sell pi coins in Pakistan at the best rate.how do i sell pi coins in Pakistan at the best rate.
how do i sell pi coins in Pakistan at the best rate.
 
how do I get a legit pi buyer in the internet (2024)
how do I get a legit pi buyer in the internet (2024)how do I get a legit pi buyer in the internet (2024)
how do I get a legit pi buyer in the internet (2024)
 
NO1 Popular Best Rohani Amil In Lahore Kala Ilam In Lahore Kala Jadu Amil In ...
NO1 Popular Best Rohani Amil In Lahore Kala Ilam In Lahore Kala Jadu Amil In ...NO1 Popular Best Rohani Amil In Lahore Kala Ilam In Lahore Kala Jadu Amil In ...
NO1 Popular Best Rohani Amil In Lahore Kala Ilam In Lahore Kala Jadu Amil In ...
 
Consumer rights and its importance and value
Consumer rights and its importance and valueConsumer rights and its importance and value
Consumer rights and its importance and value
 
Economics - Development 01 _ Handwritten Notes.pdf
Economics - Development 01 _ Handwritten Notes.pdfEconomics - Development 01 _ Handwritten Notes.pdf
Economics - Development 01 _ Handwritten Notes.pdf
 
Zepto Case study(On Track to Profitability).pptx
Zepto Case study(On Track to Profitability).pptxZepto Case study(On Track to Profitability).pptx
Zepto Case study(On Track to Profitability).pptx
 
Bitcoin Masterclass TechweekNZ v3.1.pptx
Bitcoin Masterclass TechweekNZ v3.1.pptxBitcoin Masterclass TechweekNZ v3.1.pptx
Bitcoin Masterclass TechweekNZ v3.1.pptx
 
how can I sell my pi coins in the United States at the best price
how can I sell my pi coins in the United States at the best pricehow can I sell my pi coins in the United States at the best price
how can I sell my pi coins in the United States at the best price
 
Population Growth and Economic Development
Population Growth and  Economic DevelopmentPopulation Growth and  Economic Development
Population Growth and Economic Development
 
What is an ecosystem in crypto .pdf
What  is  an  ecosystem  in  crypto .pdfWhat  is  an  ecosystem  in  crypto .pdf
What is an ecosystem in crypto .pdf
 
State Space Tutorial.pptxjjjjjjjjjjjjjjj
State Space Tutorial.pptxjjjjjjjjjjjjjjjState Space Tutorial.pptxjjjjjjjjjjjjjjj
State Space Tutorial.pptxjjjjjjjjjjjjjjj
 
Chapter Three Interest rates in the Financial System.ppt
Chapter Three Interest rates in the Financial System.pptChapter Three Interest rates in the Financial System.ppt
Chapter Three Interest rates in the Financial System.ppt
 
How can I sell my Pi coins in Vietnam easily?
How can I sell my Pi coins in Vietnam easily?How can I sell my Pi coins in Vietnam easily?
How can I sell my Pi coins in Vietnam easily?
 
Juspay Case study(Doubling Revenue Juspay's Success).pptx
Juspay Case study(Doubling Revenue Juspay's Success).pptxJuspay Case study(Doubling Revenue Juspay's Success).pptx
Juspay Case study(Doubling Revenue Juspay's Success).pptx
 
How to exchange my pi coins on HTX in 2024
How to exchange my pi coins on HTX in 2024How to exchange my pi coins on HTX in 2024
How to exchange my pi coins on HTX in 2024
 
GROUP 6 DUBAI.pptx basta amoa na dira dapita
GROUP 6 DUBAI.pptx basta amoa na dira dapitaGROUP 6 DUBAI.pptx basta amoa na dira dapita
GROUP 6 DUBAI.pptx basta amoa na dira dapita
 
how can I sell my pi coins in China 2024.
how can I sell my pi coins in China 2024.how can I sell my pi coins in China 2024.
how can I sell my pi coins in China 2024.
 
is pi Network coin available for sale in 2024
is pi Network coin available for sale in 2024is pi Network coin available for sale in 2024
is pi Network coin available for sale in 2024
 
Monthly Market Risk Update: May 2024 [SlideShare]
Monthly Market Risk Update: May 2024 [SlideShare]Monthly Market Risk Update: May 2024 [SlideShare]
Monthly Market Risk Update: May 2024 [SlideShare]
 

Test bank for advanced assessment interpreting findings and formulating differential diagnoses 4th edition - Copy.pdf

  • 1. TEST BANK FOR Advanced Assessment Interpreting Findings and Formulating Differential Diagnoses 4th Edition Goolsby Chapters 1 - 22 | Complete
  • 2. TABLE OF CONTENTS  Chapter 1. Assessment and Clinical Decision Making: An Overview  Chapter 2. Genomic Assessment: Interpreting Findings and Formulating Differential Diagnoses  Chapter 3. Skin  Chapter 4. Head, Face, and Neck  Chapter 5. The Eye  Chapter 6. Ear, Nose, Mouth, and Throat  Chapter 7. Cardiac and Peripheral Vascular Systems  Chapter 8. Respiratory System  Chapter 9. Breasts  Chapter 10. Abdomen  Chapter 11. Genitourinary System  Chapter 12. Male Reproductive System  Chapter 13. Female Reproductive System  Chapter 14. Musculoskeletal System  Chapter 15. Neurological System  Chapter 16. Nonspecific Complaints  Chapter 17. Psychiatric Mental Health  Chapter 18. Pediatric Patients  Chapter 19. Pregnant Patients  Chapter 20. Assessment of the Transgender or Gender Diverse Adult  Chapter 21. Older Patients  Chapter 22. Persons With Disabilities
  • 3.  Chapter 1. Assessment and Clinical Decision Making: An Overview Multiple Choice Identify the choice that best completesthe statement or answers the question. 1. Which type of clinical decision-making is most reliable? A. Intuitive B. Analytical C. Experiential D. Augenblick 2. Which of the following is false? To obtain adequate history, health-care providers must be: A. Methodical and systematic B. Attentive to the patient’s verbal and nonverbal language C. Able to accurately interpret the patient’s responses D. Adept at reading into the patient’s statements 3. Essential parts of a health historyinclude all of the following except: A. Chief complaint B. Historyof the present illness C. Current vital signs D. All of the above are essential history components 4. Which of the following is false? While performing the physical examination, the examiner must be able to: A. Differentiate between normal and abnormal findings B. Recall knowledge of a range of conditions and their associated signs and symptoms C. Recognize how certain conditions affect the response to other conditions D. Foresee unpredictable findings 5. The following is the least reliable source of information for diagnostic statistics: A. Evidence-based investigations B. Primaryreports of research C. Estimation based on a provider’s experience D. Published meta-analyses 6. The following can be used to assist in sound clinical decision-making: A. Algorithm published in a peer-reviewed journal article B. Clinical practice guidelines C. Evidence-based research D. All of the above 7. If a diagnostic study has high sensitivity, this indicates a: A. High percentage of persons with the given condition will have an abnormal result B. Low percentage of persons with the given condition will have an abnormal result C. Low likelihood of normal result in persons without a given condition D. None of the above 8. If a diagnostic study has high specificity, this indicates a: A. Low percentage of healthy individuals will show a normal result B. High percentage of healthyindividuals will show a normal result C. High percentage of individuals with a disorder will show a normal result D. Low percentage of individuals with a disorder will show an abnormal result 9. Alikelihood ratio above 1 indicates that a diagnostic test showing a: A. Positive result is strongly associated with the disease B. Negative result is strongly associated with absence of the disease C. Positive result is weakly associated with the disease D. Negative result is weakly associated with absence of the disease
  • 4. 10. Which of the following clinical reasoning tools is defined as evidence-based resource based on mathematical modeling to express the likelihood of a condition in select situations, settings, and/or patients?
  • 5. A. Clinical practice guideline B. Clinical decision rule C. Clinical algorithm Chapter 1: Clinical reasoning, differential diagnosis, evidence-based practice, and symptom ana Answer Section MULTIPLE CHOICE 1. ANS: B Croskerry (2009) describes two major types of clinical diagnostic decision-making: intuitive and analytical. Intuitive decision- making (similar to Augenblink decision-making) is based on the experience and intuition of the clinician and is less reliable and paired with fairly common errors. In contrast, analytical decision-making is based on careful consideration and has greater reliability with rare errors. PTS: 1 2. ANS: D To obtain adequate history, providers must be well organized, attentive to the patient’s verbal and nonverbal language, and able to accurately interpret the patient’s responses to questions. Rather than reading into the patient’s statements, they clarify any areas of uncertainty. PTS: 1 3. ANS: C Vital signs are part of the physical examination portion of patient assessment, not part of the health history. PTS: 1 4. ANS: D While performing the physical examination, the examiner must be able to differentiate between normal and abnormal findings, recall knowledge of a range of conditions, including their associated signs and symptoms, recognize how certain conditions affect the response to other conditions, and distinguish the relevance of varied abnormal findings. PTS: 1 5. ANS: C Sources for diagnostic statistics include textbooks, primary reports of research, and published meta-analyses. Another source of statistics, the one that has been most widelyused and available for application to the reasoning process, is the estimation based on a provider’s experience, although these are rarely accurate. Over the past decade, the availability of evidence on which to base clinical reasoning is improving, and there is an increasing expectation that clinical reasoning be based on scientific evidence. Evidence-based statistics are also increasingly being used to develop resources to facilitate clinical decision-making. PTS: 1 6. ANS: D To assist in clinical decision-making, a number of evidence-based resources have been developed to assist the clinician. Resources, such as algorithms and clinical practice guidelines, assist in clinical reasoning when properly applied. PTS: 1 7. ANS: A The sensitivity of a diagnostic study is the percentage of individuals with the target condition who show an abnormal, or positive, result. A high sensitivity indicates that a greater percentage of persons with the given condition will have an abnormal result. PTS: 1 8. ANS: B The specificity of a diagnostic study is the percentage of normal, healthy individuals who have a normal result. The greater the specificity, the greater the percentage of individuals who will have negative, or normal, results if they do not have the target condition. PTS: 1 9. ANS: A The likelihood ratio is the probability that a positive test result will be associated with a person who has the target condition and a negative result will be associated with a healthy person. A likelihood ratio above 1 indicates that a positive result is associated with the disease; a likelihood ratio less than 1 indicates that a negative result is associated with an absence of the disease.
  • 6. PTS: 1 10. ANS: B Clinical decision (or prediction) rules provide another support for clinical reasoning. Clinical decision rules are evidence-based resources that provide probabilistic statements regarding the likelihood that a condition exists if certain variables are met with regard to the prognosis of patients with specific findings. Decision rules use mathematical models and are specific to certain situations, settings, and/or patient characteristics. PTS: 1
  • 7. Chapter 2. Evidence-based health screening Multiple Choice Identify the choice that best completes the statement or answers the question. 1. The first step in the genomic assessment of a patient is obtaining information regarding: A. Family history B. Environmental exposures C. Lifestyle and behaviors D. Current medications 2. An affected individual who manifests symptoms of a particular condition through whom a family with a genetic disorder is ascertained is called a(n): A. Consultand B. Consulband C. Index patient D. Proband 3. An autosomal dominant disorder involves the: A. X chromosome B. Y chromosome C. Mitochondrial DNA D. Non-sex chromosomes 4. To illustrate a union between two second cousin family members in a pedigree, draw: A. Arrows pointing tothe male and female B. Brackets around the male and female C. Double horizontal lines between the male and female D. Circles around the male and female 5. To illustrate two family members in an adoptive relationship in a pedigree: A. Arrows are drawn pointing to the male and female B. Brackets are drawn around the male and female C. Double horizontal lines are drawn between the male and female D. Circles are drawn around the male and female 6. When analyzing the pedigree for autosomal dominant disorders, it is common to see: A. Several generations of affected members B. Many consanguineous relationships C. More members of the maternal lineage affected than paternal D. More members of the paternal lineage affected than maternal 7. In autosomal recessive (AR) disorders, individuals need: A. Onlyone mutated gene on the sex chromosomes to acquire the disease B. Onlyone mutated gene to acquire the disease C. Two mutated genes to acquire the disease D. Two mutated genes to become carriers 8. In autosomal recessive disorders, carriers have: A. Two mutated genes; one from each parent that cause disease B. A mutation on a sex chromosome that causes a disease C. Asingle gene mutation that causes the disease D. One copyof a gene mutation but not the disease 9. With an autosomal recessive disorder, it is important that parents understand that if theyboth carry a mutation, the following are the risks to each of their offspring (each pregnancy): A. 50% chance that offspring will carry the disease B. 10% chance of offspring affected bydisease
  • 8. 10. A woman with an X-linked dominant disorder will: C. 25% chance children will carrythe disease D. 10% chance children will be disease free A. Not be affected bythe disorder herself B. Transmit the disorder to 50 % of her offspring (male or female) C. Not transmit the disorder to her daughters D. Transmit the disorder to onlyher daughters 11. In creating your female patient’s pedigree, you note that she and both of her sisters were affected by the same genetic disorder. Although neither of her parents had indications of the disorder, her paternal grandmother and her paternal grandmother’s two sisters were affected by the same condition. This pattern suggests: A. Autosomal dominant disorder B. Chromosomal disorder C. Mitochondrial DNA disorder D. X-linked dominant disorder 12. A woman affected with an X-linked recessive disorder: A. Has one X chromosome affected bythe mutation B. Will transmit the disorder to all of her children C. Will transmit the disorder to all of her sons D. Will not transmit the mutation to anyof her daughters 13. Which of the following are found in an individual with aneuploidy? A. An abnormal number of chromosomes B. An X-linked disorder C. Select cells containing abnormal-appearing chromosomes D. An autosomal recessive disorder 14. The pedigree of a family with a mitochondrial DNA disorder is unique in that: A. None of the female offspring will have the disease B. All offspring from an affected female will have disease C. None of the offspring of an affected female will have the disease D. All the offspring from an affected male will have disease 15. Which population is at highest risk for the occurrence of aneuploidy in offspring? A. Mothers younger than 18 B. Fathers younger than 18 C. Mothers over age 35 D. Fathers over age 35 16. Approximately what percentage of cancers is due to a single-gene mutation? A. 50% to 70% B. 30% to 40% C. 20% to 25% D. 5% to 10% 17. According to the Genetic Information Nondiscrimination Act (GINA): A. NPs should keep all genetic information of patients confidential B. NPs must obtain informed consent prior to genetic testing of all patients C. Employers cannot inquire about an employee’s genetic information D. All of the above 18. The leading causes of death in the United States are due to: A. Multifactorial inheritance B. Single gene mutations C. X-linked disorders D. Aneuploidy 19. Which of the following would be considered a “red flag” that requires more investigation in a patient assessment? A. Colon cancer in family member at age 70
  • 9. 20. When patients express variable forms of the same hereditarydisorder, this is due to: B. Breast cancer in family member at age 75 C. Myocardial infarction in familymember at age 35 D. All of the above A. Penetrance B. Aneuploidy C. De novo mutation D. Sporadic inheritance 21. Your 2-year-old patient shows facial features, such as epicanthal folds, up-slanted palpebral fissures, single transverse palmar crease, and a low nasal bridge. These arereferred to as: A. Variable expressivity related to inherited disease B. Dysmorphic features related to genetic disease C. De novo mutations of genetic disease D. Different penetrant signs of genetic disease 22. In order to provide a comprehensive genetic history of a patient, the NP should: A. Ask patients to complete a family history worksheet B. Seek out pathology reports related to the patient’s disorder C. Interview family members regarding genetic disorders D. All of the above 1. 2. Evidence-based health screening Answer Section MULTIPLE CHOICE 1. ANS: A A critical first step in genomic assessment, including assessment of risk, is the use of family history. Family history is considered the first genetic screen (Berry & Shooner 2004) and is a critical component of care because it reflects shared genetic susceptibilities, shared environment, and common behaviors (Yoon, Scheuner, & Khoury2003). PTS: 1 2. ANS: D A proband is defined as the affected individual who manifests symptoms of a particular condition through whom a family with a genetic disorder is ascertained (Pagon et al. 1993–2013). The proband is the affected individual that brings the family to medical attention. PTS: 1 3. ANS: D Autosomal dominant (AD) inheritance is a result of a gene mutation in one of the 22 autosomes. PTS: 1 4. ANS: C A consanguineous family is related by descent from a common ancestry and is defined as a “union between two individuals who are related as second cousins or closer” (Hamamy 2012). Consanguinity, if present in the family history, is portrayed using two horizontal lines to establish the relationship between the male and female partners. PTS: 1 5. ANS: B For adopted members of the family, use brackets as the appropriate standardized pedigree symbol ([e.g., brackets]). PTS: 1 6. ANS: A
  • 10. Pedigrees associated with autosomal dominant (AD) disorders typically reveal multiple affected family members with the disease or syndrome. When analyzing the pedigree for AD disorders or syndromes, it is common to see a “vertical” pattern denoting several generations of affected members. PTS: 1 7. ANS: C In autosomal recessive (AR) disorders, the offspring inherits the condition by receiving one copy of the gene mutation from each of the parents. Autosomal recessive disorders must be inherited through both parents (Nussbaum et al. 2007). Individuals who have an AR disorder have two mutated genes, one on each locus of the chromosome. Parents of an affected person are called carriers because each carries one copy of the mutation on one chromosome and a normal gene on the other chromosome. Carriers typically are not affected bythe disease. PTS: 1 8. ANS: D Individuals who have an AR disorder have two mutated genes, one on each allele of the chromosome. Parents of an affected person are called carriers because each parent carries one copy of the mutation on one chromosome and a normal gene on the other chromosome. Carriers typically are not affected by the disease. In pedigrees with an AR inheritance patterns, males and females will be equally affected because the gene mutation is on an autosome. PTS: 1 9. ANS: A It is important that parents understand that if they both carry a mutation, the risk to each of their offspring (each pregnancy) is an independent event: 25% disease free, 25% affected, and 50% carrier. PTS: 1 10. ANS: B Everyone born with an X-linked dominant disorder will be affected with the disease. Transmission of the disorder to the next generation varies bygender, however. A woman will transmit the mutation to 50% of all her offspring (male or female). PTS: 1 11. ANS: D A man with an X-linked dominant disorder will transmit the mutation to 100% of his daughters (they receive his X chromosome) and none of his sons (they receive his Y chromosome). The pedigree of a family with an X-linked dominant disorder would reveal all the daughters and none of the sons affected with the disorder if the father has an X-linked disorder. PTS: 1 12. ANS: C An X-linked recessive disorder means that in a woman, both X chromosomes must have the mutation if she is to be affected. Because males have onlyone copy of the X chromosome, they will be affected if their X chromosome carries the mutation. PTS: 1 13. ANS: A An individual with an abnormal number of chromosomes has a condition called aneuploidy, which is frequently associated with mental problems or physical problems or both (Jorde, Carey, & Bamshad 2010; Nussbaum et al. 2007). PTS: 1 14. ANS: B Mitochondrial DNA is inherited from the ovum and, therefore, from the mother. The pedigree of a family with a mitochondrial DNA disorder is unique in that all offspring (regardless of gender) of an affected female will have the disease, and none of the offspring from an affected male will have the disease. PTS: 1 15. ANS: C Some individuals or couples have unique identifiable risks that should be discussed prior to conception whenever possible. For example, women who will be 35 years of age or older at delivery (advanced maternal age) are at increased risk for aneuploidy. PTS: 1 16. ANS: D The majority of cancers are sporadic or multifactorial due to a combination of genetic and environmental factors; however, approximately5% to 10% of all cancers are due to a single-gene mutation (Garber & Offit 2005).
  • 11. PTS: 1 17. ANS: D On May 21, 2008, President George W. Bush signed the Genetic Information Nondiscrimination Act (GINA) to protect Americans against discrimination based upon their genetic information when it comes to health insurance and employment, paving the way for patient personalized genetic medicine without fear of discrimination (National Human Genome Research Institute 2012). PTS: 1 18. ANS: A Most disease-causing conditions are not due to a single-gene disorder but are due to multifactorial inheritance, a result of genomics and environmental or behavioral influences. In fact, the leading causes of mortality in the United States—heart disease, cerebrovascular disease, diabetes, and cancer—are all multifactorial. Most congenital malformation, hypertension, arthritis, asthma, obesity, epilepsy, Alzheimer’s, and mental health disorders are also multifactorial. PTS: 1 19. ANS: C Early onset cancer syndromes, heart disease, or dementia are red flags that warrant further investigation regarding hereditary disorders. PTS: 1 20. ANS: A Some disorders have a range of expression from mild to severe. This variability is referred to as the penetrance of genetic disease. For example, patients with neurofibromatosis (NF1), an AD disorder of the nervous system, may manifest with many forms of the disease. For instance, some patients with NF1 may have mild symptoms, like café-au-lait spots or freckling on the axillary or skin, while others may have life-threatening spinal cord tumors or malignancy (Jorde, Carey, & Bamshad 2010; Nussbaum et al. 2007). PTS: 1 21. ANS: B Assessing for dysmorphic features may enable identification of certain syndromes or genetic or chromosomal disorders (Jorde, Carey, & Bamshad 2010; Prichard & Korf 2008). Dysmorphology is defined as “the study of abnormal physical development” (Jorde, Carey, & Bamshad 2010, 302). PTS: 1 22. ANS: D Asking the patient to complete a family history worksheet prior to the appointment saves time in the visit while offering the patient an opportunity to contribute to the collection of an accurate family history. Reviewing the family information can also help establish family rapport while verifying medical conditions in individual family members. If a hereditary condition is being considered but family medical information is unclear or unknown, requesting medical records and pathology or autopsy reports may be warranted. PTS: 1
  • 12. Chapter 3. Abdomen Multiple Choice Identify the choice that best completes the statement or answers the question. 1. When performing abdominal assessment, the clinician should perform examination techniques in the following order: A. Inspection, palpation, percussion, and auscultation B. Inspection, percussion, palpation, and auscultation C. Inspection, auscultation, percussion, and palpation D. Auscultation, palpation, percussion, and inspection 2. The clinician should auscultate the abdomen to listen for possible bruits of the: A. Aorta B. Renal artery C. Iliac artery D. All of the above 3. On abdominal examination, which of the following is assessed using percussion? A. Liver B. Kidneys C. Pancreas D. Esophagus 4. In abdominal assessment, a digital rectal examination is performed to assess for: A. Hemorrhoids B. Prostate size C. Blood in stool D. Ureteral stenosis 5. Rebound tenderness of the abdomen is a sign of: A. Constipation B. Peritoneal inflammation C. Elevated venous pressure D. Peritoneal edema 6. While assessingthe abdomen, the clinician deeplypalpates the left lower quadrant of the abdomen, and this causes pain in the patient’s right lower abdomen. This is most commonlyindicative of: A. Constipation B. Diverticulitis C. Appendicitis D. Hepatitis 7. Your patient complains of severe right lower quadrant abdominal pain. To assess the patient for peritoneal inflammation, the examiner should: A. Percuss the right lower quadrant of the abdomen B. Deeplypalpate the right lower quadrant of the abdomen C. Auscultate the right lower quadrant for hyperactive bowel sounds D. Strike the plantar surface of the patient’s heel while the patient is supine 8. Your patient is lying supine and you ask him to raise his leg while you place resistance against the thigh. The examiner is testing the patient for: A. Psoas sign B. Obturator sign C. Rovsing’s sign D. Murphys’ sign 9. A patient is lying supine and the clinician deeplypalpates the right upper quadrant of the abdomen while the patient inhales. The examiner is testing the patient for: A. Psoas sign B. Obturator sign C. Rovsing’s sign
  • 13. 10. Your patient has abdominal pain, and it is worsened when the examiner rotates the patient’s right hip inward with the knee bent and the obturator internus muscle is stretched. This is a sign of: A. Diverticulitis B. Cholecystitis C. Appendicitis D. Mesenteric adenitis 11. On abdominal examination as the clinician presses on the right upper quadrant to assess liver size, jugular vein distension becomes obvious. Hepatojugular reflux is indicative of: A. Acute hepatitis B. Right ventricular failure C. Cholecystitis D. Left ventricular failure 12. Your patient demonstrates positive shifting dullness on percussion of the abdomen. This is indicative of: A. Cholecystitis B. Appendicitis C. Ascites D. Hepatitis 13. Your 44-year-old female patient complains of right upper quadrant pain. Her skin and sclera are yellow, and she has hyperbilirubinemia and elevated liver enzymes. The clinician should suspect: A. Acute pancreatitis B. Biliary duct obstruction C. Acute hepatitis D. Atypical appendicitis 14. The most common cause of acute pancreatitis is: A. Trauma B. Hepatitis virus A C. Hyperlipidemia D. Alcohol abuse 15. Your patient with pancreatitis has a Ranson rule score of 8. The clinician should recognize that this is arisk of: A. Pleural involvement B. Alcoholism C. High mortality D. Bile duct obstruction 16. Your patient complains of left upper quadrant pain, fever, extreme fatigue, and spontaneous bruising. The clinician should recognize that these symptoms are often related to: A. Hematopoetic disorders B. Hepatomegaly C. Esophageal varices D. Pleural effusion 17. A 16-year-old patient presents with sore throat, cervical lymphadenopathy, fever, extreme fatigue, and left upper quadrant pain. The physical examination reveals splenomegaly. The clinician should recognize the probability of: A. Bacterial endocarditis B. Infectious mononucleosis C. Pneumonia with pleural effusion D. Pancreatic cancer 18. Your patient complains of lower abdominal pain, anorexia, extreme fatigue, unintentional weight loss of 10 pounds in last 3 weeks, and you find a positive hemoccult on digital rectal examination. Laboratory tests show iron deficiency anemia. The clinician needs to consider: A. Diverticulitis B. Appendicitis C. Colon cancer D. Murphys’ sign
  • 14. 19. Which of the following is the most common cause of heartburn-type epigastric pain? A. Decreased lower esophageal sphincter tone B. Helicobacteria pylori infection of stomach C. Esophageal spasm D. Excess use of NSAIDs 20. A 22-year-old female enters the emergency room with complaints of right lower quadrant abdominal pain, which has been worsening over the last 24 hours. On examination of the abdomen, there is a palpable mass and rebound tenderness over the right lower quadrant. The clinician should recognize the importance of: A. Digital rectal examination B. Endoscopy C. Ultrasound D. Pelvic examination 21. The major sign of ectopic pregnancy is: A. Sudden onset of severe epigastric pain B. Amenorrhea with unilateral lower quadrant pain C. Lower back and rectal pain D. Palpable abdominal mass 22. When ruptured ectopic pregnancyis suspected, the following procedure is most important: A. Culdocentesis B. CT scan C. Abdominal x-ray D. Digital rectal examination 23. The majorityof colon cancers are located in the: A. Transverse colon B. Cecum C. Rectosigmoid region D. Ascending colon 24. The following symptom(s) in the patient’s history should raise the clinician’s suspicion of colon cancer: A. Alternating constipation and diarrhea B. Narrowed caliber of stool C. Hematochezia D. All of the above 25. A patient presents tothe emergencydepartment with nausea and severe, colickyback pain that radiates into the groin. When asked to locate the pain, he points to the right costovertebral angle region. His physical examination is unremarkable. Which of the following lab tests is most important for the diagnosis? A. Urinalysis B. Serum electrolyte levels C. Digital rectal exam D. Lumbar x-ray 26. Your 34-year-old female patient complains of a feeling of “heaviness” in the right lower quadrant, achiness, and bloating. On pelvic examination, there is a palpable mass in the right lower quadrant. Urine and serum pregnancy tests are negative. The diagnostic tool that would be most helpful is: A. Digital rectal exam B. Transvaginal ultrasound C. Pap smear D. Urinalysis 27. Your 54-year-old male patient complains of a painless “lump” in his lower left abdomen that comes and goes for the past couple of weeks. When examining the abdomen, you should have the patient: A. Lie flat and take a deep breath B. Stand and bear down against your hand C. Prepare for a digital rectal examination D. Peptic ulcer disease
  • 15. 28. A nurse practitioner reports that your patient’s abdominal x-raydemonstrates multiple air-fluid levels in the bowel. This is a diagnostic finding found in: A. Appendicitis B. Cholecystitis C. Bowel obstruction D. Diverticulitis 29. A 76-year-old patient presents to the emergency department with severe left lower quadrant abdominal pain, diarrhea, and fever. On physical examination, you note the patient has a positive heel strike, and left lower abdominal rebound tenderness. These are typical signs and symptoms of which of the following conditions? A. Diverticulitis B. Salpingitis C. Inflammatory bowel disease D. Irritable bowel syndrome 30. Which of the following conditions is the most common cause of nausea, vomiting, and diarrhea? A. Viral gastroenteritis B. Staphylococcal food poisoning C. Acute hepatitis A D. E.coli gastroenteritis 31. A patient presents tothe emergencydepartment with complaints of vomiting and abdominal pain. You note that the emesis contains bile. On physical examination, there is diffuse tenderness, abdominal distension, and rushing, high-pitched bowel sounds. Which of the following diagnoses would be most likely? A. Gastric outlet obstruction B. Small bowel obstruction C. Distal intestinal blockage D. Colonic obstruction 32. Your 5-year-old female patient presents to the emergency department with sore throat, vomiting, ear ache, 103 degree fever, photophobia, and nuchal rigidity. She has an episode of projectile vomiting while you are examining her. The clinician should recognize that the following should be done: A. Abdominal x-ray B. Fundoscopic examination C. Lumbar puncture D. Analysis of vomitus 33. A9-year-old boyaccompanied byhis mother reports that since he came home from summer camp, he has had fever, nausea, vomiting, severe abdominal cramps and watery stools that contain blood and mucus. The clinician should recognize the importance of: A. Stool for ova and parasites B. Abdominal x-ray C. Stool for clostridium D. Fecal occult blood test 34. A 56-year-old male complains of anorexia, changes in bowel habits, extreme fatigue, and unintentional weight loss. At times he is constipated and other times he has episodes of diarrhea. His physical examination is unremarkable. It is important for the clinician to recognize the importance of: A. CBC with differential B. Stool culture and sensitivity C. Abdominal x-ray D. Colonoscopy 35. A 20-year-old engineering student complains of episodes of abdominal discomfort, bloating, and episodes of diarrhea. The symptoms usually occur after eating, and pain is frequently relieved with bowel movement. She is on a “celiac diet” and the episodic symptoms persist. Physical examination and diagnostic tests are negative. Colonoscopy is negative for any abnormalities. This is a history and physical consistent with: A. Inflammatory bowel disease B. Irritable bowel syndrome D. Lie in a left lateral recumbent position
  • 16. C. Laxative abuse D. Norovirus gastroenteritis 36. A 78-year-old female patient is suffering from heart failure, GERD, diabetes, and depression. She presents with complaints of frequent episodes of constipation. Her last bowel movement was 1 week ago. Upon examination, you palpate a hard mass is the left lower quadrant of the abdomen. You review her list of medications. Which of the following of her medications cause constipation? A. Digitalis (Lanoxin) B. Amlodipine (Norvasc) C. Sertraline (Zoloft) D. Metformin (Glucophage) 37. You are examining a 55-year-old female patient with a historyof alcohol abuse. She complains of anorexia, nausea, pruritus, and weight loss over the last month. On physical examination, you note yellow hue of the skin and sclera. Which of the following physical examination techniques is most important? A. Scratch test B. Heel strike C. Digital rectal examination D. Pelvic examination 38. You observe Charcot’s triad of sign and symptoms in a patient under your care. This is commonlyseen in which of the following disorders? A. Cirrhosis B. Pancreatitis C. Cholangitis D. Portal hypertension 39. A 59-year-old patient with history of alcohol abuse is admitted for hematemesis. On physical examination, you note ascites and caput medusa. Alikelycause for the hematemesis is: A. Peptic ulcer disease B. Barrett’s esophagus C. Pancreatitis D. Esophageal varices 40. A 16-year-old female with anorexia and bulimia is admitted for hematemesis. She admits to inducing vomiting often. On physical examination, you note pallor, BMI less than 15, and hypotension. Alikelyreason for hematemesis is: A. Mallory-Weiss tear B. Cirrhosis C. Peptic ulcer disease D. Esophageal varies 41. An 82-year-old female presents to the emergencydepartment with epigastric pain and weakness. She admits to having dark, tarry stools for the last few days. She reports a long history of pain due to osteoarthritis. She self-medicates daily with ibuprofen, naprosyn, and aspirin for joint pain. On physical examination, she has orthostatic hypotension and pallor. Fecal occult blood test is positive. A likely etiology of the patient’s problem is: A. Mallory-Weiss tear B. Esophageal varices C. Gastric ulcer D. Colon cancer 42. A 48-year-old male presents to the clinic with complaints of anorexia, nausea, weakness, and unintentional weight loss over the last few weeks. On physical examination, the patient has jaundice of the skin as well as sclera and a palpable mass in the epigastric region. In addition to CBC and bilirubin levels, all of the following tests would be helpful except: A. Liver enzymes B. Amylase C. Lipase D. Uric acid 43. Your 66-year-old male patient complains of weakness, fatigue, chronic constipation for the last month, and dark stools. On CBC, his results show iron deficiency anemia. Colon cancer is diagnosed. Which of the following laboratory tests is used to follow progress of colon cancer?
  • 17. A. Alpha fetoprotein (AFP) B. Carcinogenic embryonic antigen (CEA) C. Carcinoma antigen 125 (CA-125) D. Beta-human chorionic gonadotropin (beta HCG) 44. Your patient is a 33-year-old female gave birth last week. She complains of constipation, rectal pain, and itching. She reports bright red blood on the toilet tissue. The clinician should recognize the need for: A. Digital rectal exam B. CEA blood test C. Colonoscopy D. Fecal occult blood test Chapter 3. Abdomen Answer Section MULTIPLE CHOICE 1. ANS: C The abdominal examination begins with inspection, followed by auscultation, percussion, and palpation. Light palpation should precede deep palpation. Auscultating before percussion or palpation allows the examiner to listen to the abdominal sounds undisturbed. Moreover, if pain is present, it is best to leave palpation until last and to gather other data before possibly causing the patient discomfort. PTS: 1 2. ANS: D Perform auscultation before palpation so as to hear unaltered bowel sounds. Listen for bruits over the aorta and the iliac, renal, and femoral arteries. PTS: 1 3. ANS: A The purpose of liver percussion is to measure the liver size. The technique used to percuss theliver is as follows: 1. Starting in the midclavicular line at about the 3rd intercostal space, lightly percuss and move down. 2. Percuss inferiorlyuntil dullness denotes the liver's upper border (usuallyat fifth intercostal space inMCL). 3. Resume percussion from below the umbilicus on the midclavicular line in an area of tympany. 4. Percuss superiorlyuntil dullness indicates the liver's inferior border. 5. Measure span in centimeters. Normal liver span: clinically estimated at midclavicular line: 6-12 cm and midsternal line: 4-8 cm. PTS: 1 4. ANS: D A digital rectal examination is included in the abdominal examination. Note skin changes or lesions in the perianal region or the presence of external hemorrhoids. Insert the gloved index finger into the anus with the patient either leaning over or side-lying on the examination table, and note any internal hemorrhoids or fissures. Check the stool for occult blood. For males, the rectal examination is necessary for direct examination of the prostate. Ureteral stenosis is detected by angiographt. PTS: 1 5. ANS: B Rebound tenderness is tested by slowly pressing over the abdomen with your fingertips, holding the position until pain subsides or the patient adjusts to the discomfort, and then quickly removing the pressure. Rebound pain, a sign of peritoneal inflammation, is present if the patient experiences a sharp discomfort over the inflamed site when pressure is released. PTS: 1 6. ANS: C Appendicitis is suggested by a positive Rovsing’s sign. This sign is positive when there is referred rebound pain in the right lower quadrant when the examiner presses deeplyin the left lower quadrant and then quickly releases the pressure. PTS: 1 7. ANS: D Ask the patient to stand with straight legs and to raise up on toes. Then ask the patient to relax, allowing the heel to strike the floor, thus jarring the body. A positive heel strike is indicative of appendicitis and peritoneal irritation. Alternatively, strike the plantar surface of the heel with your fist while the patient rests supine on the examination table.
  • 18. PTS: 1 8. ANS: A To examine the patient for appendicitis, the clinician can test the patient for psoas sign. This is done in the following manner: Place a hand on the patient’s thigh just above the knee and ask the patient to raise the thigh against your hand. This contracts the psoas muscle and produces pain in patients with an inflamed appendix. PTS: 1 9. ANS: D Murphy’s Sign is elicited by deeply palpating the right upper quadrant of the abdomen. Pain is present on deep inspiration when an inflamed gallbladder is palpated by pressing the fingers under the rib cage. Murphy’s sign is positive in cholecystitis. PTS: 1 10. ANS: C A positive obturator sign indicates appendicitis. Pain is elicited by inward rotation of the right hip with the knee bent so that the obturator internus muscle is stretched. PTS: 1 11. ANS: B Hepatojugular reflux is elicited by applying firm, sustained hand pressure to the abdomen in the midepigastric region while the patient breathes regularly. Observe the neck for elevation of the jugular venous pressure (JVP) with pressure of the hand and a sudden drop of the JVP when the hand pressure is released. Hepatojugular reflux is exaggerated in right heart failure. PTS: 1 12. ANS: C To assess the patient for ascites, test for shifting of the peritoneal fluid to the dependent side by rolling the patient side to side and percussing for dullness on the dependent side of the abdomen. PTS: 1 13. ANS: B In cholecystitis, acute colicky pain is localized in the RUQ and is often accompanied by nausea and vomiting. Murphy’s sign is frequently present. Fever is low grade, and the increase in neutrophilic leukocytes in the blood is slight. Acute cholecystitis improves in 2 to 3 days and resolves within a week; however, recurrences are common. If acute cholecystitis is accompanied by jaundice and cholestasis (arrest of bile excretion), suspect common duct obstruction. PTS: 1 14. ANS: D Biliary tract disease and alcoholism account for 80% or more of the pancreatitis admissions. Other causes include hyperlipidemia, drugs, toxins, infection, structural abnormalities, surgery, vascular disease, trauma, hyperparathyroidism and hypercalcemia, renal transplantation, and hereditary pancreatitis. The most common cause of pancreatitis is alcohol abuse. PTS: 1 15. ANS: C The Ranson rule uses a score determined by MRI results, with an index possible range of 0 to 10. A categorization of patients indicates the risk of both mortality and complication from pancreatitis. Patients at the low end of the index (1–3) are predicted to have a low risk of mortality (3%) and complications (8%), whereas patients scoring at the high end (7–10) of the index are predicted to have a higher incidence of mortality(17%) and/or complications (92%). PTS: 1 16. ANS: A LUQ pain can be associated with stomach or spleen disorders; however, it is often associated with causes that are outside the abdomen. Hematopoietic malignancies, such as lymphomas and leukemias, and other hematologic disorders, such as thrombocytopenia, polycythemia, myelofibrosis, and hemolyticanemia, often cause enlargement of the spleen, leading to LUQ pain. In addition to questions about the specific characteristics of the pain, it is important to ask the patient about fever, unusual bleeding or bruising, recent diagnosis of mononucleosis, fatigue, malaise, lymphadenopathy, cough, arthralgias, anorexia, weight loss, jaundice, high blood pressure, and headache. PTS: 1 17. ANS: B Hypersplenism is secondary to other primary disorders, most commonly cytopenic hematologic disorders, such as lymphoma, leukemia, thrombocytopenia, polycythemia, myelofibrosis, and haemolytic anemias. With the sore throat and cervical
  • 19. lymphadenopathy, infection due to Epstein-Barr virus is common in adolescents. Infectious mononucleosis is an important disorder to consider. Splenomegaly often occurs in infectious mononucleosis. PTS: 1 18. ANS: C A positive hemoccult on rectal examination may indicate an upper GI bleed or malignancy. Malignancy should also be suspected if there is weight loss and/or a palpable abdominal mass. PTS: 1 19. ANS: A GERD is the most common organic cause of heartburn. GERD is caused by decreased lower esophageal sphincter (LES) tone. LES control can be decreased by several medications (e.g., theophylline, dopamine, diazepam, calcium-channel blockers), foods and/or beverages (caffeine, alcohol, chocolate, fatty foods), and tobacco use. When LES tone is lower than normal, secretions are allowed to reflux into the esophagus, causing discomfort. PTS: 1 20. ANS: D A female with abdominal pain can have a GI or GU disorder or gynecologic problem. It is imperative to ask about the last menstrual period (LMP) and about birth control methods in order to rule out ectopic pregnancy. A history of miscarriages and/or sexually transmitted diseases (STDs) can give more clues for the risk of ectopic pregnancy. Safe sex practices and the number of sexual partners can alert the practitioner to the risk for pelvic inflammatory disease. No complaint of lower abdominal pain in a female should be evaluated without performing a pelvic examination. PTS: 1 21. ANS: B The most obvious sign of ectopic pregnancy is amenorrhea followed by spotting and sudden onset of severe lower quadrant pain. A stat pregnancy test should be performed. There is tenderness on pelvic examination, and a pelvic mass may be palpated. Blood is present in the cul-de-sac. Shock and hemorrhage occur if the pregnancy ruptures. Abdominal distention with peritoneal signs will ensue. Immediate laparoscopy or laparotomy is indicated because this condition is life threatening. PTS: 1 22. ANS: A The most obvious sign of ectopic pregnancy is amenorrhea followed by spotting and sudden onset of severe lower quadrant pain. A stat pregnancy test should be performed. The diagnosis of ectopic pregnancy can be made with urine human chorionic gonadotropin (hCG) or stat serum hCG, pelvic ultrasound, and, if necessary, culdocentesis to detect blood in the cul-de-sac. There is tenderness on pelvic examination, and a pelvic mass may be palpated. Blood is present in the cul-de-sac. Shock and hemorrhage occur if the pregnancy ruptures. Abdominal distension with peritoneal signs will ensue. Immediate laparoscopy or laparotomy is indicated because this condition is life threatening. PTS: 1 23. ANS: C Colorectal cancer is the second leading cause of death from malignancies in the United States. Over half are located in the rectosigmoid region and are typically adenocarcinomas. Risk factors include a history of polyps, positive family history of colon cancer or familial polyposis, ulcerative colitis, granulomatous colitis, and a diet low in fiber and high in animal protein, fat, and refined carbohydrates. PTS: 1 24. ANS: D Colon cancer may be present for several years before symptoms appear. Complaints include fatigue, weakness, weight loss, alternating constipation and diarrhea, a change in the caliber of stool, tenesmus, urgency, and hematochezia. Physical examination is usually normal except in advanced disease, when the tumor can be palpated or hepatomegaly is present, owing to metastatic disease. PTS: 1 25. ANS: A Urinary calculi can occur anywhere in the urinary tract; therefore, pain can originate in the flank or kidney area and radiate into the RLQ or LLQ and then to the suprapubic area as the stone attempts to move down the tract. The pain is severe, acute, and colicky and may be accompanied by nausea and vomiting. If the stone becomes lodged at the ureterovesical junction, the patient will complain of urgency and frequency. Blood will be present in the urine. PTS: 1
  • 20. 26. ANS: B Ovarian masses are often asymptomatic, but symptoms may include pressure-type pain, heaviness, aching, and bloating. Masses are typically detected on pelvic examination. In advanced malignancies, ascites is often present. An elevated cancer antigen 125 (CA-125) result indicates the likelihood that the mass is malignant. A transvaginal pelvic ultrasound has a higher diagnostic sensitivity than transabdominal ultrasound. If diagnosis is unclear, CT, MRI, or PET scan can be performed. A laparoscopy or exploratory laparotomy is necessary for staging, tumor debulking, and resection. PTS: 1 27. ANS: B In the majority of hernia cases, a history of heavy physical labor or heavy lifting can be elicited. Right or left lower quadrant pain that may radiate into the groin or testicle is typical. The pain is usually dull or aching unless strangulated, in which case the pain is more severe. The pain increases with straining, lifting, or movement of the lower extremities. Physical examination includes palpating the femoral area and inguinal ring for bulging or tenderness. Ask the patient to bear down against your hand. PTS: 1 28. ANS: C The most common causes of mechanical obstruction are adhesions, almost exclusively in patients with previous abdominal surgery, hernias, tumors, volvulus, inflammatory bowel disease (Crohn’s disease, colitis), Hirschsprung’s disease, fecal impaction, and radiation enteritis. Initially, the patient complains of a cramping periumbilical pain that eventually becomes constant. Physical examination reveals mild, diffuse tenderness without peritoneal signs, and possibly visible peristaltic waves. In early obstruction, tinkles, rushes, and borborygmi can be heard. In late obstruction, bowel sounds may be absent. The diagnosis can be made with flat and upright abdominal films looking for bowel distension and the presence of multiple air-fluid levels. CT or MRI may be necessary for confirmation. PTS: 1 29. ANS: A Diverticular disease is prevalent in patients over 60 years of age. Since the sigmoid colon has the smallest diameter of any portion of the colon, it is the most common site for the development of diverticula. Although the pain can be generalized, it is typically localized to the left lower abdomen and is accompanied by tenderness, fever, and leukocytosis. Other symptoms can include constipation or loose stools, nausea, vomiting, and positive stool occult blood. With diverticulitis, there is an increased risk of perforation, which presents with a more dramatic clinical picture as a result of peritonitis. Look for signs of peritonitis, such as a positive heel strike test and/or rebound tenderness. PTS: 1 30. ANS: A Viral gastroenteritis is the most common cause of nausea, vomiting, and diarrhea. At least 50% of cases of gastroenteritis as foodborne illness are due to norovirus. Another 20% of cases, and the majority of severe cases in children, are due to rotavirus. Other significant viral agents include adenovirus and astrovirus. PTS: 1 31. ANS: B The contents of the vomitus commonly vary according to the level of obstruction. Gastric outlet obstruction is associated with emesis containing undigested food. Proximal small intestinal blockage is likely to be bile-stained. Distal intestinal blockage is more likely to contain fecal matter. The degree of cramping and pain is often related to the proximity of the obstruction, so that obstructions of the lower intestines may have less severe cramping, vomiting, and/or pain. Bowel sounds often are high pitched and metallic sounding but may later become absent. Tenderness may be localized or diffuse. Distention as well as a succussion splash may be present. PTS: 1 32. ANS: C The range of neurologic disorders that result in nausea and/or vomiting is broad. Included are meningitis, increased intracranial pressure (ICP), migraines, a space-occupying lesion, and Ménière’s disorder. Central nervous system-related vomiting is often projectile and may not be preceded by nausea. Papilledema may accompany increased ICP. Neurological deficits may be evident with increased ICP, space-occupying lesions, and meningitis. Nuchal rigidity is a classic finding for meningitis. PTS: 1 33. ANS: A Parasites causing diarrhea usually enter the body through the mouth. They are swallowed and can remain in the intestine or burrow through the intestinal wall and invade other organs. Certain parasites, most commonly Giardia lamblia, transmitted by fecally contaminated water or food, can cause diarrhea, bloating, flatulence, cramps, nausea, anorexia, weight loss, greasy stools because of its interference with fat absorption, and occasionallyfever. Symptoms usually occur about 2 weeks after exposure and
  • 21. can last 2 to 3 months. Often, the symptoms are vague and intermittent, which makes diagnosis more difficult. Serial stool samples for O&P should be ordered because a single sample may not reveal the offending parasite. PTS: 1 34. ANS: D The symptoms and severity of the diarrhea vary according to the underlying cause. The symptoms of carcinomas are generally insidious. The diarrhea is mild and intermittent. Often malignancies are found on routine hemoccults, sigmoidoscopy, or colonoscopy. There should be a high index of suspicion with unexplained weight loss or new-onset iron-deficiency anemia in a patient over 40 years old. PTS: 1 35. ANS: B Irritable bowel syndrome (IBS) is a functional bowel disorder characterized by mild to severe abdominal pain, discomfort, bloating, and alteration of bowel habits. The exact cause is unknown. In some cases, the symptoms are relieved by bowel movements. Diarrhea or constipation may predominate, or they may be mixed (classified as IBS-D, IBS-C, or IBS-M, respectively). IBS may begin after an infection (postinfectious, IBS-PI) or a stressful life event. IBS is a motility disorder involving the upper and lower GI tracts that causes intermittent nausea, abdominal pain and distention, flatulence, pain relieved by defecation, diarrhea, and/or constipation. Symptoms usually occur in the waking hours and may be worsened or triggered by meals. It is three times more prevalent in women, accounts for more than half of all GI referrals, and is highly correlated with emotional factors, particularly anxiety and stress. PTS: 1 36. ANS: B Medications that frequently cause constipation include: - Analgesics/narcotics - Antacids containing aluminum - Anticonvulsants - Antidepressants - Antihypertensives (calcium-channel blockers, beta blockers) - Antiparkinsonism agents - Antispasmodics - Calcium supplements - Diuretics - Iron supplements - Sedatives/tranquilizers PTS: 1 37. ANS: A Cirrhosis develops with the replacement of normal liver tissue by regenerative, fibrotic nodules and may occur in the late phase of a variety of disorders that damage the liver, such as alcohol toxicity. A patient may present with jaundice and describe an associated, progressive pattern of pruritus, weakness, anorexia, nausea, and weight loss. Determine the size and consistency of the liver as well as any tenderness. The scratch test is a method used to ascertain the location and size of a patient's liver during a physical assessment. The scratch test uses auscultation to detect the differences in sound transmission through the abdominal cavity over solid and hollow organs and spaces. After placing a stethoscope over the approximate location of a patient's liver, the examiner will then scratch the skin of the patient's abdomen lightly, moving laterally along the liver border. When the liver is encountered, the scratching sound heard in the stethoscope will increase significantly. In this manner, the size and shape of a patient's liver can be ascertained. PTS: 1 38. ANS: C Occlusion of the common bile duct may occur with disorders of the gallbladder and/or bile duct, such as cholecystitis, cholelithiasis, and cholangitis. All three conditions are generally accompanied by RUQ discomfort, anorexia, and nausea. Charcot’s triad, which includes jaundice, RUQ pain, and fever/chills, is common to problems resulting in obstructions of the bile duct. PTS: 1 39. ANS: D Patients with portal hypertension may develop GI bleeding from varices of the esophagus, stomach, intestines, or other sites. Portal hypertension is most commonly associated with cirrhosis, usually caused by alcohol abuse or hepatitis. Check for signs of liver disease, including jaundice, cirrhosis, telangiectasia, hepatomegaly, and RUQ tenderness. Ascites occurs due to venous congestion. Caput medusa is the distension of paraumbilical veins due to portal hypertension.
  • 22. PTS: 1 40. ANS: A Upper GI hemorrhage may result from a tear at the gastroesophageal junction, known as a Mallory-Weiss tear. A patient may develop more than one tear. These tears are most common in alcoholic or bulimic patients following repeated episodes of vomiting or severe retching. If a laceration/tear of the mucosa causes GI bleeding, the patient may demonstrate alterations in hemodynamic status. PTS: 1 41. ANS: C Bleeding occurs after an area of gastric mucosal injury has ulcerated. Explore symptoms of epigastric and/or periumbilical discomfort. Identify potential causes of gastric mucosal injury—the most common being NSAID use and stress. Many elderly individuals self-medicate with over-the-counter aspirin preparations and various NSAIDs. Commonly, they use too many medications that have side effects of gastric irritation. PTS: 1 42. ANS: D Primary or metastatic cancers of the liver and/or pancreas can cause obstructive hyperbilirubinemia and jaundice. Jaundice may be the initial sign of a malignancy or may follow the development of other symptoms. Ask about associated symptoms, such as RUQ discomfort, nausea, fever, back pain, weight loss, fatigue/weakness, and pruritus. None of these symptoms are specific to malignancy; however, other causes of jaundice are less likely to be associated with weight loss. During the abdominal examination, carefully palpate the area of the liver and the remainder of the abdomen, checking for masses or unexpected findings. In addition to a CBC, liver functions, amylase, lipase, and bilirubin levels, abdominal CT and/or ultrasound should be ordered promptly. PTS: 1 43. ANS: B AFP can help diagnose and guide the treatment of liver cancer (hepatocellular carcinoma). CA-125 is the standard tumor marker used to follow women during or after treatment for epithelial ovarian cancer (the most common type of ovarian cancer) as well as fallopian tube cancer and primary peritoneal cancer. Serum beta HCG is a pregnancy marker. CEA is not used to diagnose or screen for colorectal cancer, but it’s the preferred tumor marker to help predict outlook in patients with colorectal cancer. The higher the CEA level at the time colorectal cancer is detected, the more likelyit is that the cancer is advanced. PTS: 1 44. ANS: A The most common cause of lower GI bleeding is hemorrhoids. The bleeding associated with hemorrhoids is usually evident as red blood on the formed stool, in the toilet bowl, or on the toilet tissue following a bowel movement. Patients with hemorrhoids often complain of rectal discomfort as well as the contributing factors for hemorrhoid development, including constipation. Inspect the perianal rectal tissue. Anoscopy may be indicated. Perform a digital rectal examination to assess internal haemorrhoids. PTS: 1
  • 23. Chapter 4: Affective Changes MULTIPLE CHOICE 1. When performing a physical assessment, the first technique the nurse will always use is: a. Palpation. b. Inspection. c. Percussion. d. Auscultation. ANS: B The skills requisite for the physical examination are inspection, palpation, percussion, and auscultation. The skills are performed one at a time and in this order (with the exception of the abdominal assessment, during which auscultation takes place before palpation and percussion). The assessment of each body system begins with inspection. A focused inspection takes time and yields a surprising amount of information. 2. The nurse is preparing to perform a physical assessment. Which statement is true about the physical assessment? The inspection phase: a. Usually yields little information. b. Takes time and reveals a surprising amount of information. c. Maybe somewhat uncomfortable for the expert practitioner. d. Requires a quick glance at the patients body systems before proceeding with palpation. ANS: B Afocused inspection takes time and yields a surprising amount of information. Initially, the examiner may feel uncomfortable, staring at the person without also doing something. A focused assessment is significantly more than a quick glance. 3. The nurse is assessing a patients skin during an office visit. What part of the hand and technique should be used to best assess the patients skin temperature? a. Fingertips; they are more sensitive to small changes in temperature. b. Dorsal surface of the hand; the skin is thinner on this surface than on the palms. c. Ulnar portion of the hand; increased blood supply in this area enhances temperature sens Palmar surface of the hand; this surface is the most sensitive to temperature variations b d. ANS:B of its increased nerve supply in this area. The dorsa (backs) of the hands and fingers are best for determining temperature because the skin is thinner on the dorsal surfaces than on the palms. Fingertips are best for fine, tactile discrimination. The other responses are not useful for palpation.
  • 24. lO M oA R cP S D| 1 2 2 6 3 4 2 3 4. Which of these techniques uses the sense of touch to assess texture, temperature, moisture, and swelling when the nurse is assessing a patient? a. Palpation b. Inspection c. Percussion d. Auscultation ANS: A Palpation uses the sense of touch to assess the patient for these factors. Inspection involves vision; percussion assesses through the use of palpable vibrations and audible sounds; and auscultation uses the sense of hearing. 5. The nurse is preparing to assess a patients abdomen by palpation. How should the nurse proceed? Palpation of reportedly tender areas are avoided because palpation in these areas may ca a. pain. Palpating a tender area is quickly performed to avoid any discomfort that the patient ma b. experience. The assessment begins with deep palpation, while encouraging the patient to relax and to c. deep breaths. The assessment begins with light palpation to detect surface characteristics and to accust d. ANS:D patient to beingtouched. Light palpation is initially performed to detect any surface characteristics and to accustom the person to being touched. Tender areas should be palpated last, not first. 6.The nurse would use bimanual palpation technique in which situation? a. Palpating the thorax of an infant b. Palpating the kidneys and uterus c. Assessingpulsations and vibrations d. Assessing the presence of tenderness and pain ANS: B Bimanual palpation requires the use of both hands to envelop or capture certain body parts or organs such as the kidneys, uterus, or adnexa. The other situations are not appropriate for bimanual palpation. 7. The nurse is preparing to percuss the abdomen of a patient. The purpose of the percussion is to assess the of the underlying tissue. a. Turgor b. Texture Downloaded byAnna Maina(annamurugijoe@gmail.com)
  • 25. lO M oA R cP S D| 1 2 2 6 3 4 2 3 Downloaded byAnna Maina(annamurugijoe@gmail.com) c. Density d. Consistency ANS: C Percussion yields a sound that depicts the location, size, and density of the underlying organ. Turgor and texture are assessed with palpation. 8. The nurse is reviewing percussion techniques with a newly graduated nurse. Which technique, if used by the new nurse, indicates that more review is needed? a. Percussing once over each area b. Quickly lifting the striking finger after each stroke c. Striking with the fingertip, not the finger pad d. Using the wrist to make the strikes, not the arm ANS: A For percussion, the nurse should percuss two times over each location. The striking finger should be quickly lifted because a resting finger damps off vibrations. The tip of the striking finger should make contact, not the pad of the finger. The wrist must be relaxed and is used to make the strikes, not the arm. 9. When percussing over the liver of a patient, the nurse notices a dull sound. The nurse should: a. Consider this a normal finding. b. Palpate this area for an underlying mass. c. Reposition the hands, and attempt to percuss in this area again. d. Consider this finding as abnormal, and refer the patient for additional treatment. ANS: A Percussion over relatively dense organs, such as the liver or spleen, will produce a dull sound. The other responses are not correct. 10. The nurse is unable to identify any changes in sound when percussing over the abdomen of an obese patient. What should the nurse do next? a. Ask the patient to take deep breaths to relax the abdominal musculature. b. Consider this finding as normal, and proceed with the abdominal assessment. c. Increase the amount of strength used when attempting to percuss over the abdomen. d. Decrease the amount of strength used when attempting to percuss over the abdomen. ANS: C The thickness of the persons body wall will be a factor. The nurse needs a stronger percussion stroke for persons with obese or very muscular body walls. The force of the blowdetermines the loudness of the note. The other actions are not correct. 11. The nurse hears bilateral loud, long, and low tones when percussing over the lungs of a 4-year-old child. The nurse should:
  • 26. Downloaded byAnna Maina(annamurugijoe@gmail.com) lO M oA R cP S D| 1 2 2 6 3 4 2 3 a. Palpate over the area for increased pain and tenderness. b. Ask the child to take shallow breaths, and percuss over the area again. c. Immediately refer the child because of an increased amount of air in the lungs. d. Consider this finding as normal for a child this age, and proceed with the examination. ANS: D Percussion notes that are loud in amplitude, low in pitch, of a booming quality, and long in duration are normal over a childs lung. 12. A patient has suddenly developed shortness of breath and appears to be in significant respiratory distress. After calling the physician and placing the patient on oxygen, which of these actions is the best for the nurse to take when further assessing the patient? a. Count the patients respirations. b. Bilaterally percuss the thorax, noting any differences in percussion tones. c. Call for a chest x-ray study, and wait for the results before beginning an assessment. d. Inspect the thorax for any new masses and bleeding associated with respirations. ANS: B Percussion is always available, portable, and offers instant feedback regarding changes in underlying tissue density, which may yield clues of the patients physical status. 13. The nurse is teaching a class on basic assessment skills. Which of these statements is true regarding the stethoscope and its use? a. Slope of the earpieces should point posteriorly (toward the occiput). b. Although the stethoscope does not magnify sound, it does block out extraneous room no c. Fit and quality of the stethoscope are not as important as its ability to magnify sound. d. Ideal tubing length should be 22 inches to dampen the distortion of sound. ANS: B The stethoscope does not magnify sound, but it does block out extraneous room sounds. The slope of the earpieces should point forward toward the examiners nose. Long tubing will distort sound. The fit and quality of the stethoscope are both important. 14. The nurse is preparing to use a stethoscope for auscultation. Which statement is true regarding the diaphragm of the stethoscope? The diaphragm: a. Is used to listen for high-pitched sounds. b. Is used to listen for low-pitched sounds. c. Should be lightly held against the persons skin to block out low-pitched sounds. d. Should be lightly held against the persons skin to listen for extra heart sounds and murm
  • 27. Downloaded byAnna Maina(annamurugijoe@gmail.com) lO M oA R cP S D| 1 2 2 6 3 4 2 3 ANS: A The diaphragm of the stethoscope is best for listening to high-pitched sounds such as breath, bowel, and normal heart sounds. It should be firmly held against the persons skin, firmly enough to leave a ring. The bell of the stethoscope is best for soft, low- pitched sounds such as extra heart sounds or murmurs. 15. Before auscultating the abdomen for the presence of bowel sounds on a patient, the nurse should: a. Warm the endpiece of the stethoscope by placing it in warm water. Leave the gown on the patient to ensure that he or she does not get chilled during the b. examination. c. Ensure that the bell side of the stethoscope is turned to the on position. d. Check the temperature of the room, and offer blankets to the patient if he or she feels col ANS: D The examination room should be warm. If the patient shivers, then the involuntary muscle contractions can make it difficult to hear the underlying sounds. The end of the stethoscope should be warmed between the examiners hands, not with water. The nurse should never listen through a gown. The diaphragm of the stethoscope should be used to auscultate for bowel sounds. 16. The nurse will use which technique of assessment to determine the presence of crepitus, swelling, and pulsations? a. Palpation b. Inspection c. Percussion d. Auscultation ANS: A Palpation applies the sense of touch to assess texture, temperature, moisture, organ location and size, as well as any swelling, vibration or pulsation, rigidity or spasticity, crepitation, presence of lumps or masses, and the presence of tenderness or pain. 17. The nurse is preparing to use an otoscope for an examination. Which statement is true regarding the otoscope? The otoscope: a. Is often used to direct light onto the sinuses. b. Uses a short, broad speculum to help visualize the ear. c. Is used to examine the structures of the internal ear. d. Directs light into the ear canal and onto the tympanic membrane. ANS: D
  • 28. Downloaded byAnna Maina(annamurugijoe@gmail.com) lO M oA R cP S D| 1 2 2 6 3 4 2 3 The otoscope directs light into the ear canal and onto the tympanic membrane that divides the external and middle ear. A short, broad speculum is used to visualize the nares. 18. An examiner is using an ophthalmoscope to examine a patients eyes. The patient has astigmatism and is nearsighted. The use of which of these techniques would indicate that the examination is being correctly performed? a. Using the large full circle of light when assessing pupils that are not dilated b. Rotating the lens selector dial to the black numbers to compensate for astigmatism c. Using the grid on the lens aperture dial to visualize the external structures of the eye d. Rotating the lens selector dial to bring the object into focus ANS: D The ophthalmoscope is used to examine the internal eye structures. It can compensate for nearsightedness or farsightedness, but it will not correct for astigmatism. The grid is used to assess size and location of lesions on the fundus. The large full spot of light is used to assess dilated pupils. Rotating the lens selector dial brings the object into focus. 19. The nurse is unable to palpate the right radial pulse on a patient. The best action would be to: a. Auscultate over the area with a fetoscope. b. Use a goniometer to measure the pulsations. c. Use a Doppler device to check for pulsations over the area. d. Check for the presence of pulsations with a stethoscope. ANS: C Doppler devices are used to augment pulse or blood pressure measurements. Goniometers measure joint range of motion. A fetoscope is used to auscultate fetal heart tones. Stethoscopes are used to auscultate breath, bowel, and heart sounds. 20. The nurse is preparing to perform a physical assessment. The correct action by the nurse is reflected by which statement? The nurse: a. Performs the examination from the left side of the bed. b. Examines tender or painful areas first to help relieve the patients anxiety. c. Follows the same examination sequence, regardless of the patients age or condition. d. Organizes the assessment to ensure that the patient does not change positions too often. ANS: D The steps of the assessment should be organized to ensure that the patient does not change positions too often. The sequence of the steps of the assessment may differ, depending on the age of the person and the examiners preference. Tender or painful areas should be assessed last.
  • 29. Downloaded byAnna Maina(annamurugijoe@gmail.com) lO M oA R cP S D| 1 2 2 6 3 4 2 3 21. A man is at the clinic for a physical examination. He states that he is very anxious about the physical examination. What steps can the nurse take to make him more comfortable? a. Appear unhurried and confident when examininghim. b. Stay in the room when he undresses in case he needs assistance. c. Ask him to change into an examining gown and to take off his undergarments. Defer measuring vital signs until the end of the examination, which allows him time to b d. ANS:A comfortable. Anxiety can be reduced by an examiner who is confident, self-assured, considerate, and unhurried. Familiar and relatively nonthreatening actions, such as measuring the persons vital signs, will gradually accustom the person to the examination. 22. When performing a physical examination, safety must be considered to protect the examiner and the patient against the spread of infection. Which of these statements describes the most appropriate action the nurse should take when performing a physicalexamination? Washing ones hands after removing gloves is not necessary, as long as the gloves are sti a. intact. b. Hands are washed before and after everyphysical patient encounter. Hands are washed before the examination of each body system to prevent the spread of c. bacteria from one part of the body to another. Gloves are worn throughout the entire examination to demonstrate to the patient concern d. ANS:B regardingthe spread of infectious diseases. The nurse should wash his or her hands before and after every physical patient encounter; after contact with blood, body fluids, secretions, and excretions; after contact with any equipment contaminated with body fluids; and after removing gloves. Hands should be washed after gloves have been removed, even if the gloves appear to be intact. Gloves should be worn when potential contact with any body fluids is present. 23. The nurse is examining a patients lower leg and notices a draining ulceration. Which of these actions is most appropriate in this situation? a. Washinghands, and contactingthe physician b. Continuing to examine the ulceration, and then washing hands c. Washing hands, putting on gloves, and continuing with the examination of the ulceration Washinghands, proceeding with rest of the physical examination, and then continuing w d. examination of the leg ulceration
  • 30. Downloaded byAnna Maina(annamurugijoe@gmail.com) lO M oA R cP S D| 1 2 2 6 3 4 2 3 ANS: C The examiner should wear gloves when the potential contact with any body fluids is present. In this situation, the nurse should wash his or her hands, put on gloves, and continue examining the ulceration. 24. During the examination, offering some brief teaching about the patients body or the examiners findings is often appropriate. Which one of these statements by the nurse is most appropriate? a. Your atrial dysrhythmias are under control. b. Youhave pitting edema and mild varicosities. c. Your pulse is 80 beats per minute, which is within the normal range. d. Imusing my stethoscope to listen for any crackles, wheezes, or rubs. ANS: C The sharing of some information builds rapport, as long as the patient is able to understand the terminology. 25. The nurse keeps in mind that the most important reason to share information and to offer brief teaching while performing the physical examination is to help the: a. Examiner feel more comfortable and to gain control of the situation. b. Examiner to build rapport and to increase the patients confidence in him or her. c. Patient understand his or her disease process and treatment modalities. d. Patient identify questions about his or her disease and the potential areas of patient educ ANS: B Sharing information builds rapport and increases the patients confidence in the examiner. It also gives the patient a little more control in a situation during which feeling completely helpless is often present.
  • 31. Downloaded byAnna Maina(annamurugijoe@gmail.com) lO M oA R cP S D| 1 2 2 6 3 4 2 3 Chapter 5: Amenorrhea 1. If amenorrhea is reported by the female patient, the clinician should attribute this to until proven otherwise. A. Pregnancy B. Ovarian failure C. Lack of progesterone D. Menopause 2. A 17-year-old female patient complains of amenorrhea for the last 8 months, weight gain, excessive hair growth on the arms and chest, and development of acne. Pelvic examination is normal. The clinician should recognize these are signs of: A. Ovarian cancer B. Endometriosis C. Hormonal imbalance D. Stress-induced anovulation 3.. A 55-year-old woman complains of amenorrhea and hot flashes for the last 3 months. Pregnancy test is negative. The clinician should recognize that menopause is defined as the absence of menses for: A. 3 months B. 6 months C. 1 year D. 18 months 4. To confirm the diagnosis of amenorrhea due to menopause, blood should be drawn for hormones. Which are the changes that occur with menopause? A. FSH rises B. LH decreases C. LH rises D. A & C 1. ANS: A Last menstrual period is one of the most important questions to ask, particularly when prescribing medications, because many are contraindicated in pregnancy. If menstrual cycles are not regular, pregnancy should be ruled out first, and then other diagnoses can be considered. The menstrual history includes any episodes of amenorrhea, menorrhagia (excessive bleeding at the time of the menstrual cycle), metrorrhagia (bleeding at irregular noncyclic intervals), dysmenorrhea, and postmenopausal bleeding. Amenorrhea has many causes, including pregnancy; anorexia nervosa; excessive exercise; low body fat; and disorders or tumors of the hypothalamus, pituitary gland, ovary, uterus, and thyroid gland. 2.. ANS: C Anovulation may present with amenorrhea but also may present with dysfunctional uterine bleeding, polymenorrhea, or menorrhagia. The symptoms vary with the cause. Overweight may be seen with several of the causes, including hypothyroidism, polycystic ovarian syndrome, and pituitary and adrenal dysfunction. Underweight is seen in anorexia nervosa, excessive exercise, hyperthyroidism, or stress-induced anovulation. Hirsutism, acne, and other skin changes can be seen with imbalances in LH, FSH, and androgens, as seen in polycystic ovary disease. Delayed puberty or regression of sexual characteristics is seen in hypopituitarism; galactorrhea can be the presenting symptom in pituitary tumors. PTS: 1 3.. ANS: B Menopause is the absence of menses for at least 6 months. Age of menopause varies greatly, although age 50 to 55 is the typical range of onset. Although the absence of menses in a woman around the age of 50 years is diagnostic for menopause, measurement of FSH, LH, and estradiol levels are helpful in confirming the diagnosis. PTS: 1 4.. ANS: D In menopause, FSH rises first and then LH rises, both greater than 100 mU/mL. A fall in estradiol is the last hormonal change that occurs with the decline of ovarian function. An estradiol level of less than 30 pg/mL indicates loss of ovarian function.
  • 32. Downloaded byAnna Maina(annamurugijoe@gmail.com) lO M oA R cP S D| 1 2 2 6 3 4 2 3 Chapter 6. Breasts lumps and nipple discharge Multiple Choice Identify the choice that best completes the statement or answers the question. 1. Your patient has a rubbery, firm, mobile breast mass. In order to completely exclude the possibility of cancer, the triple test is necessary, which includes: A. CT scan B. MRI C. Biopsy D. All of the above 2. A23-year-old woman is concerned about a mass she found on palpation. Upon examination of the patient, the mass feels cystic, round, and mobile with discrete borders. The diagnostic study that is recommended is: A. CT scan B. Mammogram C. Biopsy D. Ultrasound 3. Which of the following variables is not a component of the Gail Model? A. Age at menarche B. Number of breast biopsies C. Age at first live birth D. Number of live births 4. A62-year-old female presents with a singular, hard, 1 cm, non-tender, non-mobile mass in the right breast. There are no nipple or skin changes, however, you palpate an enlarged right-sided supraclavicular lymph node. The clinician should recognize these are signs of: A. Fibroadenoma B. Breast cyst C. Malignancy D. Paget’s disease 5. A45-year-old female patient presents in the emergencydepartment due to multiple chest injuries as a result of a motor vehicle accident. There is a palpable, tender, irregular 3 cm soft mass located in the left breast, with erythema and swelling of the breast. Which of the following is indicated? A. Compression wrap for chest and breast trauma B. Mammogram C. Re-evaluation following complete resolution of obvious injuries D. Ultrasound 6. Your patient is a 46-year-old woman suffering from psychosis complains of milkydischarge from both her breasts. Upon examination, there are no significant findings and the patient is not pregnant, breastfeeding, or in the postpartum period. You should: A. Order CT scan of head to rule out pituitary tumor B. Obtain a complete list of the patient’s medications C. Have the patient keep a menstrual cycle diary D. Refer the patient for mammogram 7.. Your patient is a 36-year-old woman who complains of milky discharge from both her breasts, episodes of headache, and menstrual irregularity. The patient is on no medications. Upon examination, there are no significant findings, and the patient is not pregnant, breastfeeding, or in the postpartum period. It is important to: A. Order prolactin level B. Obtain an MRI of head C. Have the patient keep a menstrual cycle diary D. Refer the patient for mammogram
  • 33. Downloaded byAnna Maina(annamurugijoe@gmail.com) lO M oA R cP S D| 1 2 2 6 3 4 2 3 8. Bloodybreast discharge is associated with which of the following? A. Bleeding disorder such as von Willebrand disease B. Malignancy C. Excessive non-steroidal anti-inflammatories D. A and C 9. Your patient is a 64-year-old female who presents with complaints of a red, scaly rash on her nipple and areola of one breast. It has been present for a few months. There is no mass or nipple discharge. You should be suspicious of: A. Contact dermatitis B. Mastitis C. Paget’s disease D. Eczema 10. A12-year-old boyis brought into the clinic byhis mother. The teen boyis embarrassed because he has developed breast tissue and would like to know if there is any treatment. The physical examination reveals early stages of puberty, otherwise it is normal. This presentation is: A. Associated with a high rate of malignancy B. Most often due to altered hormonal levels in puberty C. Commonlyindicative of pituitary tumor triggered by puberty D. Too dense for a mammogram and requires biopsy 11 A condition that increases risk of breast cancer in a male is: A. Pseudogynecomastia B. BRCA1 and 2 C. Klinefelter’s syndrome D. B and C Chapter 6. Breasts lumps and nipple discharge Answer Section MULTIPLE CHOICE PTS: 1 1. ANS: C Palpation and mammography, alone or together, are inadequate to definitively identify the cause of a breast mass and to rule out malignancy. The “triple test” recommended for evaluation of a breast mass involves clinical examination, either ultrasound or mammogram, and aspiration and/or biopsy. The determination of whether an ultrasound or mammography is recommended is based on age and other situations. PTS: 1 2. ANS: D The American College of Radiology (2012) recommends that women under 40 years of age should be assessed through ultrasound, and women who are older than 40 years should be assessed with a mammogram. Younger women have denser breasts, and ultrasound is, therefore, often more useful than a mammogram. Ultrasounds are helpful in determining whether or not a mass that feels potentially “cystic” is fluid filled or solid. It is crucial that the woman understand that the imaging procedure is a screening tool and is never diagnostic regarding the existence or absence of a malignancy. PTS: 1 3. ANS: D The Gail model is a clinical prediction rule used to estimate a patient’s risk for breast cancer. The model identifies the relative risk associated with three factors obtained through history: age at menarche, number of previous breast biopsies, and the age at first live birth. PTS: 1 4. ANS: C The typical malignant breast mass is solitary, non-tender, hard, immobile or fixed, and poorly defined. It may be accompanied by nipple erosion or other inflammatory skin changes, as seen in Paget’s disease; nipple discharge; skin thickening or dimpling; retraction; and palpable axillary nodes. Although most malignant masses are painless, associated discomfort does not exclude the potential for breast cancer.
  • 34. Downloaded byAnna Maina(annamurugijoe@gmail.com) lO M oA R cP S D| 1 2 2 6 3 4 2 3 PTS: 1 5. ANS: B Trauma to the anterior chest area may result in a palpable breast mass. An automobile accident with injury from contact with the seat belt, air bag, steering wheel, or dashboard is a common source of breast trauma. When a palpable mass results from chest trauma, it typically represents either a hematoma or area of secondary fat necrosis. Even when a mass is identified subsequent to direct trauma, the provider must remain suspicious for the possibility of malignancy that preexisted but was undetected before the accident. PTS: 1 6. ANS: B Galactorrhea is characterized by bilateral and milky discharge from multiple ducts in a woman who is neither pregnant nor lactating. Causes of galactorrhea include a variety of drugs as well as an elevated prolactin level associated with pituitary tumor or hyperthyroidism. The drugs associated with galactorrhea include antidepressants (amitriptyline, imipramine), psychoactives (haloperidol, thioridazine), hormones (estrogens, progestogens), antiepileptics (valproic acid), and antihypertensives (verapamil). This list is not exhaustive. PTS: 1 7. ANS: A Galactorrhea can be a symptom of endocrine disorder, particularly pituitary dysfunction. In prolactinoma of the pituitary, a patient complains of galactorrhea, headaches, vision change, relative infertility, and amenorrhea. Laboratory studies should include a pregnancy test, prolactin level, and thyroid functions. PTS: 1 8. ANS: B Bloody discharge is often associated with malignancy but can stem from other conditions. If blood is not evident, the discharge should be tested for blood, using guaiac process. PTS: 1 9 ANS: C In Paget’s disease of the breast, the patient may describe the persistence of skin changes for several months. The typical presentation involves skin changes of the nipple and/or areola, with the nipple being involved first. The condition does not always involve a palpable mass or nodule. The skin changes of the nipple and areola range from scaling redness to various degrees of ulceration. PTS: 1 10. ANS: B Gynecomastia most often occurs during infancy, puberty, and senescence. It is caused by an altered balance between estradiol and testosterone levels. Although it can be an indication of primary hypogonadism, hyperthyroidism, cirrhosis, or renal disease, the majority of the cases are specific to hormonal changes of puberty, are drug induced, or are idiopathic. With the presentation of breast enlargement in a male, malignancy must always be considered. PTS: 1 11 ANS: D It is anticipated that over 2,000 new cases of male breast cancer will be diagnosed in 2013, with over 400 related deaths (ACS 2013). Men develop the same types of breast cancer as women. Therefore, it is important to include malignancy in the differential diagnosis when a man complains of breast enlargement. Factors that increase the risk of breast cancer in men include a previous history of breast or testicular disease and Klinefelter’s syndrome. A history of gynecomastia is not associated with an increased risk for breast cancer.
  • 35. Downloaded byAnna Maina(annamurugijoe@gmail.com) lO M oA R cP S D| 1 2 2 6 3 4 2 3 Chapter 7. Breast Pain Multiple Choice Identify the choice that best completes the statement or answers the question. 1. When examining the breasts of a 45-year-old female patient, you note nodularity throughout both breasts. There is no dominant mass, tenderness, nipple change, or skin change. Which of the following condition is most likely? A. Multiple fibroadenomas B. Fibrocystic breasts C. Mastalgia D. Mastitis 2. A 23-year-old female presents with episodicbilateral breast tenderness. Upon palpation, there are multiple areas of nodularitywith no dominant breast mass and no tenderness, nipple, or skin changes. It is best torecommend: A. Ultrasound of the breasts B. Mammogram of the breasts C. Fine needle aspiration of one nodule within a breast D. Diaryof menstrual cycle and breast symptoms 3.. A 34-year-old female who gave birth 2 weeks ago developed fever and pain in the right breast while nursing her baby. The patient has tenderness, eythema, and swelling of the nipple on the right breast. Which of the following diagnostic studies is indicated? A. White blood cell (WBC) count B. Breast milk culture C. Ultrasound D. Fine needle biopsy 4.. A 52-year-old female complains of discharge from one of her breasts. There is no pain, no mass, and no skin changes. The physical examination is normal. When putting pressure on the affected breast, the nipple expresses a small amount of thick, white discharge. These findings are consistent with: A. Duct ectasia B. Mastitis C. Fibroadenoma D. Fibrocystic breasts Chapter 7. Breast Pain Answer Section MULTIPLE CHOICE 1. ANS: A Breast tissue is normally glandular and may have a rather nodular consistency. The degree of nodularity tends to fluctuate through the menstrual cycle in premenopausal women. A dominant breast mass is a mass that persists throughout a woman’s hormonal cycles, is larger and firmer than any other irregularities, and differs from rest of the breast tissue. A fibroadenoma is a soft discrete breast mass that is moveable, non-tender, and benign. Mastalgia is a condition of pain in the breast. Mastitis is an infection that causes erythema, swelling, and discharge from the breast. PTS: 1 2.. ANS: D Women who experience cyclic mastalgia usually have onset as a teen or young adult. It is important to determine menstrual and reproductive history and to identify all pharmacologic agents taken. A complete breast examination should be performed. The pain associated with hormonal fluctuation most commonly occurs during the second half of the woman’s cycle. The variability of the signs and symptoms is identified with a symptom diary and record of the menstrual cycle. The pain is typically poorly localized, bilateral, and nonspecific. It may be accompanied by a sense of breast fullness. The examination may identify the multiple, bilateral nodularities associated with fibroadenomas or fibrocystic changes. PTS: 1
  • 36. Downloaded byAnna Maina(annamurugijoe@gmail.com) lO M oA R cP S D| 1 2 2 6 3 4 2 3 3. ANS: A For mastitis, white blood count should be obtained and is usually elevated. Even though the breast milk can be cultured, this is not generally recommended. If the presentation is atypical—that is, the patient is not lactating—and there are no associated systemic signs or symptoms, a consultation should be obtained and mammography ordered to determine the definitive diagnosis and rule out malignancy. PTS: 1 4.. ANS: A Discharge from one breast is commonly benign and frequently caused by duct ectasia or an intraductal papilloma. Duct ectasia results in dilation of one major breast duct and causes approximately one-third of the cases of pathological discharge. Papillomas are responsible for 44% of pathologic discharge. Because the discharge associated with ductal ectasia is often stagnant, the discharge is cheesy in appearance. It is often associated with noncyclic breast discomfort and a subareolar lump at the site of the dilated duct. Because one major duct is involved, the discharge comes also from one duct or nipple area. PTS: 1
  • 37. Downloaded byAnna Maina(annamurugijoe@gmail.com) lO M oA R cP S D| 1 2 2 6 3 4 2 3 Chapter 8: Chest Pain 1. A. Cardiac chest pain is most often described as: Stabbing, piercing pain B. Pain with inhalation C. Crushing, squeezing pain D. Burning, gnawing pain 2. A. The pain associated with pericarditis is . Crushing and squeezing B. Constant C. Worse with inspiration D. Onlypresent with fever 3. Pain associated with a dissecting thoracic aortic aneurysm is commonlydescribed as: A. Retrosternal crushing and squeezing B. Chest stabbing and sharp C. Ripping and tearing in the chest or thoracic back D. Worse with inspiration 4. The pain of can frequentlybe mistaken for cardiac chest pain. A. Gastroesophageal reflux disease (GERD) B. Peptic ulcer disease (PUD) C. Cholecystitis D. All of the above 5. The pain of pancreatitis is described as: A. Abdominal sharp and piercing pain in the left upper quadrant B. Dull and cramping pain in the right upper quadrant C. Severe, epigastric pain radiating straight into the back D. Sharp pain radiating to the shoulder 6. The pain of costochondritis typically . A. Mimics cardiac crushing and squeezing pain B. Worsens with movement and full inspiration C. Radiates from epigastrium into the back D. Is a tearing and ripping pain 7. Patients with anxiety frequently complain of: A. Chest pain B. Dizziness C. Shortness of breath D. A and B 8. . During a cardiac assessment on a 38-year-old patient in the hospital for chest pain, the nurse finds the following: jugular vein pulsations 4 cm above the sternal angle when the patient is elevated at 45 degrees, blood pressure 98/60 mm Hg, heart rate 130 beats per minute, ankle edema, difficultybreathing when supine, and an S3 on auscultation. Which of these conditions best explains the cause of these findings? a. Fluid overload
  • 38. Downloaded byAnna Maina(annamurugijoe@gmail.com) lO M oA R cP S D| 1 2 2 6 3 4 2 3 b. Atrial septal defect c. MI d. Heart failure ANS: D 1. ANS: C Typical characteristics that indicate acute coronary syndrome (ACS) include crushing, squeezing substernal chest pain with radiation to the neck or left arm, a score of greater than 7 on the pain scale, an association with exertion or stress with relief on rest, a duration of minutes, and associated symptoms of nausea, diaphoresis, weakness, or shortness of breath. 2. ANS: C Unlike the symptoms associated with ACS, the pain associated with pericarditis is sharp and stabbing; it may worsen with inspiration or when lying flat or leaning forward. Associated symptoms may include shortness of breath, fever, chills, and malaise. PTS: 1 3. ANS: C Aortic aneurysms are often asymptomatic. However, in a dissecting aortic aneurysm , symptoms are often described as tearing or ripping in the chest, back, or abdomen. PTS: 1 4. ANS: D It is often difficult to differentiate the symptoms of gastroesophageal reflux disease (GERD) or peptic ulcer disease (PUD) from cardiac symptoms. A thorough history and diagnostic tests are necessary. Patients with a history of GERD or PUD should still be worked up for a cardiac etiology, particularly if the characteristics of the symptoms or the history have changed to raise the index of suspicion for cardiac disease. The pain of cholecystitis, also sometimes mistaken for cardiac pain, typically presents with right upper quadrant pain with radiation to the thoracic region of the back. PTS: 1 5. ANS: C The pain of pancreatitis is severe, steady, and “boring”—radiating from the epigastric region through to the back. It is often accompanied by nausea and vomiting, tachycardia, hypotension, and diaphoresis. These symptoms are also seen in MI; however, the exquisite abdominal tenderness present in pancreatitis assists in differentiating it from cardiac pain. PTS: 1 6. ANS: B Costochondritis, which is inflammatory pain of the chest wall, can often be differentiated from cardiac pain through history. A history of injury, heavy lifting, contact sports, excessive coughing, or late-stage pregnancy (which stretches the intercostal muscles) leads the examiner to consider chest wall pain. This often occurs in a younger population with no cardiac risk factors. One of the most helpful differentiating symptoms is that the pain is increased with movement, cough, or, in some cases, respiration. 7. ANS: D More than 90% of patients with anxiety present primarily with somatic complaints in primary care and emergency department settings (Stern & Herman, 2003). Patients may initially complain of only somatic symptoms before they are ultimately diagnosed with a primary anxiety disorder (DeVane et al., 2005). Patients with anxiety often present with the following symptoms: chest pain (with negative angiogram), irritable bowel, unexplained dizziness, migraine headache, and chronic fatigue. 8. Ans. D.Heart failure causes decreased cardiac output when the heart fails as a pump and the circulation becomes backed up and congested. Signs and symptoms include dyspnea, orthopnea, paroxysmal nocturnal dyspnea, decreased blood pressure,
  • 39. Downloaded byAnna Maina(annamurugijoe@gmail.com) lO M oA R cP S D| 1 2 2 6 3 4 2 3 dependent and pitting edema; anxiety; confusion; jugular vein distention; and fatigue. The S3 is associated with heart failure and is always abnormal after 35 years of age. The S3 maybe the earliest sign of heart failure.
  • 40. Downloaded byAnna Maina(annamurugijoe@gmail.com) lO M oA R cP S D| 1 2 2 6 3 4 2 3 Chapter 9. Confusion in older adults Multiple Choice Identify the choice that best completes the statement or answers the question. 1. An 85-year-old female patient arrives by ambulance to the emergency department accompanied by her husband. He reports that his wife had been ill with pneumonia and, 2 days ago, went to the family physician who prescribed azithromycin twice a day. The husband reports that he is making sure she gets the medicine. His wife has been staying in bed and resting. She awoke from sleep last night and was extremely agitated, left the house, and was walking outside. She did not recognize her husband and wanted to call the police. Which of the following is an appropriate question for the history? A. Does your wife have dementia or frequent episodes of confusion? B. Has your wife been running a fever? C. Is your wife allergic to any medication? D. Has your wife ever had a mental status exam? PTS: 1 ANS: A Delirium can be observed in both elderly and younger patients and is generally defined as an acute confusional state, affecting all aspects of cognition and mentation. The signs and symptoms of delirium generally have a more acute or rapidly progressive onset as opposed to the slow, gradual decline noted in the organic dementias. The acute mental status change is often associated with other signs or symptoms—such as hallucinations, illusions, incoherent speech, and constant aimless activity—that help to narrow the differential diagnosis. Electrolyte disturbances, infection, and polypharmacy are frequent causes of delirium in the elderly. 2. An older patient has been admitted to the intensive care unit (ICU) after falling at home. Within 8 hours, his condition has stabilized and he is transferred to a medical unit. The family is wondering whether he will be able to go back home. Which assessment instrument is most appropriate for the nurse to choose at this time? a. Lawton IADL instrument b. Hospital Admission Risk Profile (HARP) c. Mini-Cog d. NEECHAM Confusion Scale ANS: B Hospital-acquired functional decline may occur within 2 days of a hospital admission. The HARP helps identify older adults who are at greatest risk of losing their ability to perform ADLs or mobility at this critical time. The Lawton IADL measures instrumental activities of daily living, which may be difficult to observe in the hospital setting. The Mini-Cog is an assessment of mental status. The NEECHAM Confusion Scale is used to assess for delirium.
  • 41. Downloaded byAnna Maina(annamurugijoe@gmail.com) lO M oA R cP S D| 1 2 2 6 3 4 2 3 3. During a functional assessment of an older persons home environment, which statement or question by the nurse is most appropriate regarding common environmental hazards? a. These low toilet seats are safe because they are nearer to the ground in case of falls. b. Do you have a relative or friend who can help to install grab bars in your shower? c. These small rugs are ideal for preventing you from slipping on the hard floor. d. It would be safer to keep the lighting low in this room to avoid glare in your eyes. ANS: B Environmental hazards within the home can be a potential constraint on the older persons day-to- day functioning. Common environmental hazards, including inadequate lighting, loose throw rugs, curled carpet edges, obstructed hallways, cords in walkways, lack of grab bars in tub and shower, and low and loose toilet seats, are hazards that could lead to an increased risk of falls and fractures. Environmental modifications can promote mobility and reduce the likelihood of the older adult falling. 4. When assessing aging adults, the nurse knows that one of the first things that should be assessed before making judgments about their mental status is: a. Presence of phobias b. General intelligence c. Presence of irrational thinking patterns d. Sensory-perceptive abilities ANS: D Age-related changes in sensoryperception can affect mental status. For example, vision loss (as detailed in Chapter 14) may result in apathy, social isolation, and depression. Hearing changes are common in older adults, which produces frustration, suspicion, and social isolation and makes the person appear confused. 5. During the taking of the health history of a 78-year-old man, his wife states that he occasionally has problems with short-term memory loss and confusion: He cant even remember how to button his shirt. When assessing his sensory system, which action by the nurse is most appropriate? a.The nurse would not test the sensory system as part of the examination because the results would not be valid. b.The nurse would perform the tests, knowing that mental status does not affect sensory ability. c.The nurse would proceed with an explanation of each test, making certain that the wife understands. d.Before testing, the nurse would assess the patients mental status and ability to follow directions.